SlideShare une entreprise Scribd logo
1  sur  150
Télécharger pour lire hors ligne
Midterm I Review
                                       Math 21a


                                     March 5, 2008


        .


.
Image: Flickr user Mr. Theklan
                                                     .   .   .   .   .   .
Announcements




    ◮   Midterm covers up to Section 11.4 in text
    ◮   Any topics not covered in class or on HW will not be on test
        (i.e., curvature)
    ◮   Complete list of learning objectives on the Midterm I Review
        sheet
    ◮   Odd problems, chapter reviews, old exams, reviews




                                                 .   .    .    .   .   .
Outline

   Vectors and the Geometry of           Derivatives and Integrals of
   Space                                 Vector Functions
      Three-Dimensional                  Arc Length (not Curvature)
      Coordinate Systems                 Motion in Space: Velocity and
      Vectors                            Acceleration
      The Dot Product                    Parametric Surfaces
      The Cross Product               Partial Derivatives
      Equations of Lines and Planes      Functions of Several Variables
      Functions and surfaces             Utility Functions and
      Cylindrical and Spherical          indifference curves
      Coordinates                        Limits and Continuity
   Vector Functions                      Partial Derivatives
      Vector Functions and Space         Tangent Planes and Linear
      Curves                             Approximations

                                                .    .    .    .   .      .
Three-Dimensional Coordinate Systems
Section 9.1 Learning Objectives



        ◮   To understand and be able to apply rectangular coordinate
            systems in R3 and the right hand rule.
        ◮   To understand and be able to find the distance d between two
            points (x1 , y1 , z1 ) and (x2 , y2 , z2 ) in R3 and to be able to use the
            distance formula
                               √
                        d = (x1 − x2 )2 + (y1 − y2 )2 + (z1 − z2 )2 .

        ◮   To understand and be able to apply the equation of a sphere of
            radius r and center (x0 , y0 , z0 ),

                          (x − x0 )2 + (y − y0 )2 + (z − z0 )2 = r2 .



                                                            .    .     .    .     .      .
Axes in three-dimensional space
                 z
                 .                .
                                  z


                                              y
                                              .

                 .
                          y
                          .                           x
                                                      .
                                                  x
                                                  .
         x
         .
                 y
                 .
                                                          .
                                                          y




                              x
                              .
                                  z
                                  .


         .
         z
                                      .   .   .               .   .   .
The right-hand rule

                       z
                       .




               x
               .

                           .

                   .
                   y




          .


                               .   .   .   .   .   .
Distance in space

   Example
   Find the distance between the points P1 (3, 2, 1) and P2 (4, 4, 4).




                                                    .    .    .    .     .   .
Distance in space

   Example
   Find the distance between the points P1 (3, 2, 1) and P2 (4, 4, 4).
                                  z
                                  .
   Solution
                                                     ..2
                                                      P

                                                 .
                                                 d   .
                                                     3

                                      .        2
                                               .               .
                                                               y
                                      1
                                      ..     √
                                      P
                                      .1     . 5

                          x
                          .
                              √              √
                        d=        5 + 32 =       1+4+9

                                                     .     .       .   .   .   .
Distance in space—General
   Theorem
   The distance between (x1 , y1 , z1 ) and (x2 , y2 , z2 ) is
                   √
                     (x2 − x1 )2 + (y2 − y1 )2 + (z2 − z1 )2

   In space, the locus of all points which are a fixed distance from a
   fixed point is a sphere.




                                                      .    .     .   .   .   .
Munging an equation to see its surface
   Example
   Find the surface is represented by the equation

                 x2 + y2 + z2 − 4x + 8y − 10z + 36 = 0




                                                 .   .   .   .   .   .
Munging an equation to see its surface
   Example
   Find the surface is represented by the equation

                   x2 + y2 + z2 − 4x + 8y − 10z + 36 = 0


   Solution
   We can complete the square:

   0 = x2 − 4x + 4 + y2 + 8y + 16 + z2 − 10z + 25 + 36 − 4 − 16 − 25
     = (x − 2)2 + (y + 4)2 + (z − 5)2 − 9

   So

                      (x − 2)2 + (y + 4) + (z − 5)2 = 9
                             =⇒ |(x, y, z)(2, −4, 5)| = 3

   This is a sphere of radius 3, centered at (2, −4, 5).
                                                       .   .   .   .   .   .
Vectors
Learning objectives for Section 9.2



        ◮   To understand and be able to apply the definition of a scalar and
            a vector.
        ◮   To be able to represent vectors geometrically as directed line
            segments or algebraically as ordered pairs or triples of numbers.
        ◮   To understand and be able to apply the axioms or vector
            addition and scalar multiplication.
        ◮   To be understand and be able to determine the length of a
            vector.
        ◮   To understand the definitions of unit vectors, standard basis
            vectors, and position vectors and to be able to apply these
            definitions.


                                                       .    .    .    .    .    .
What is a vector?
   Definition
    ◮   A vector is something that has magnitude and direction
    ◮   We denote vectors by boldface (v) or little arrows (⃗ One is
                                                            v).
        good for print, one for script
    ◮   Given two points A and B in flatland or spaceland, the vector
        which starts at A and ends at B is called the displacement
                −→
        vector AB.
    ◮   Two vectors are equal if they have the same magnitude and
        direction (they need not overlap)

                                 B
                                 .                    D
                                                      .
                           .
                           v                  .
                                              u


                    ..
                    A                  C
                                       .
                                                  .       .   .   .    .   .
Vector or scalar?



   Definition
   A scalar is another name for a real number.

   Example
   Which of these are vectors or scalars?
    (i) Cost of a theater ticket
    (ii) The current in a river
   (iii) The initial flight path from Boston to New York
   (iv) The population of the world




                                                 .    .   .   .   .   .
Vector or scalar?



   Definition
   A scalar is another name for a real number.

   Example
   Which of these are vectors or scalars?
    (i) Cost of a theater ticket scalar
    (ii) The current in a river vector
   (iii) The initial flight path from Boston to New York vector
   (iv) The population of the world scalar




                                                 .   .    .      .   .   .
Vector addition

   Definition
   If u and v are vectors positioned so the initial point of v is the
   terminal point of u, the sum u + v is the vector whose initial point is
   the initial point of u and whose terminal point is the terminal point
   of v.

                                                      .
                                                      u
                              .
                              v                           .
                                                          v
                      . +v
                      u                      . +v
                                             u
                                         .
                                         v
                     .
                     u                       .
                                             u
               .                      .
               The triangle law       The parallelogram law


                                                  .       .   .   .   .      .
Opposite and difference

   Definition
   Given vectors u and v,
     ◮   the opposite of v is the vector −v that has the same length as
         v but points in the opposite direction
     ◮   the difference u − v is the sum u + (−v)


                                              .
                                              v


                                   .
                                   u
                              .         −
                                        . v
                            . −v
                            u


                                                  .   .   .    .   .      .
Scaling vectors

   Definition
   If c is a nonzero scalar and v is a vector, the scalar multiple cv is
   the vector whose
     ◮   length is |c| times the length of v
     ◮   direction is the same as v if c > 0 and opposite v if c < 0
   If c = 0, cv = 0.


                                          2
                                          .v
                                    .
                                    v
                        −2
                        . 1v   .




                                                   .    .    .    .    .   .
Components defined
  Definition
    ◮    Given a vector a, it’s often useful to move the tail to O and
         measure the coordinates of the head. These are called the
         components of a, and we write them like this:

                                    a = ⟨a1 , a2 , a3 ⟩

         or just two components if the vector is the plane. Note the
         angle brackets!
    ◮    Given a point P in the plane or space, the position vector of
                         −
                         →
         P is the vector OP.

  Fact                                                                    −
                                                                          →
  Given points A(x1 , y1 , z1 ) and B(x2 , y2 , z2 ) in space, the vector AB has
  components
                        −
                        →
                        AB = ⟨x2 − x1 , y2 − y1 , z2 − z1 ⟩
                                                          .    .    .     .        .   .
Vector algebra in components




   Theorem
   If a = ⟨a1 , a2 , a3 ⟩ and b = ⟨b1 , b2 , b3 ⟩, and c is a scalar, then
     ◮   a + b = ⟨a1 + b1 , a2 + b2 , a3 + b3 ⟩
     ◮   a − b = ⟨a1 − b1 , a2 − b2 , a3 − b3 ⟩
     ◮   ca = ⟨ca1 , ca2 , ca3 ⟩




                                                           .    .     .      .   .   .
Properties



   Theorem
   Given vectors a, b, and c and scalars c and d, we have

     1.   a+b=b+a                         6. (c + d)a = ca + da
     2.   a + (b + c ) = ( a + b ) + c
                                          7. (cd)a = c(da)
     3.   a+0=a
     4.   a + (−a) = 0                    8. 1a = a
     5.   c(a + b) = ca + cb

   These can be verified geometrically.




                                                    .       .   .   .   .   .
Length



   Definition
   Given a vector v, its length is the distance between its initial and
   terminal points.

   Fact
   The length of a vector is the square root of the sum of the squares of its
   components:                            √
                        |⟨a1 , a2 , a3 ⟩| =   a2 + a2 + a2
                                               1    2    3




                                                       .     .   .    .    .    .
Useful vectors


   Definition
     ◮    A unit vector is a vector whose length is one
     ◮    We define the standard basis vectors i = ⟨1, 0, 0⟩,
          j = ⟨0, 1, 0⟩, k = ⟨0, 0, 1⟩. In script, they’re often written as ˆ, ȷ,
                                                                            ı ˆ
          ˆ
          k.

   Fact
   Any vector a can be written as a linear combination of the standard basis
   vectors
                       ⟨a1 , a2 , a3 ⟩ = a1 i + a2 j + a3 k.




                                                        .    .    .     .    .      .
The Dot Product I
Learning objectives for Section 9.3




        ◮   To understand and be able to use the definitions of the dot
            product to measure the length of a vector and the angle θ
            between two vectors. Given two vectors a = ⟨a1 , a2 , a3 ⟩ and
            b = ⟨b1 , b2 , b3 ⟩,

                         a · b = |a||b| cos θ = a1 b1 + a2 b2 + a3 b3 .

        ◮   To understand and be able to apply properties of the dot
            product.




                                                          .    .    .     .   .   .
The Dot Product II
Learning objectives for Section 9.3




        ◮   To understand that two vectors are orthogonal if their dot
            product is zero.
        ◮   To understand and be able to determine the projection of a
            vector a onto a vector b,

                                                  a·b
                                      proja b =        a
                                                  |a|2

        ◮   To be able to use vectors and the dot product in applications.




                                                           .   .   .   .   .   .
Definition
If a and b are any two vectors in the plane or in space, the dot
product (or scalar product) between them is the quantity

                           a · b = |a| |b| cos θ,

where θ is the angle between them.
Another way to say this is that a · b is |b| times the length of the
projection of a onto b.

                                            .
                                            a

          . · b is |b| times this length.
          a
                                                        .
                                                        b



In components, if a = ⟨a1 , a2 , a3 ⟩ and b = ⟨b1 , b2 , b3 ⟩, then
                       a · b = a1 b1 + a2 b2 + a3 b3.       .   .     .   .   .
Algebraic properties of the dot product




   Fact
   If a, b and c are vectors are c is a scalar, then

     1. a · a = |a|2                         4. (ca) · b = c(a · b) = a · (cb)
     2. a · b = b · a                        5. 0 · a = 0
     3. a · (b + c) = a · b + a · c




                                                       .    .   .    .    .      .
Geometric properties of the dot product


   Fact
   The projection of b onto a is given by

                                             a·b
                               proja b =          a
                                             |a|2


   Fact
   The angle between two nonzero vectors a and b is given by

                                             a·b
                                cos θ =             ,
                                            |a| |b|

   where θ is taken to be between 0 and π .


                                                        .   .   .   .   .   .
More geometric properties of the dot product




   Fact
   The angle between two nonzero vectors a and b is acute if a · b > 0.
   obtuse if a · b < 0, right if a · b = 0. The vectors are parallel if
   a · b = ± |a| |b|.
     ◮    b is a positive multiple of a if a · b = |a| |b|
     ◮    b is a negative multiple of a if a · b = − |a| |b|




                                                         .     .   .   .   .   .
Other uses of the dot product




    ◮   similarity
    ◮   Work
                          W=F·d




                                  .   .   .   .   .   .
The Cross Product
Learning objectives for Section 9.4



        ◮   To understand and be able to use the definitions of the cross
            product to find a vector that is orthogoal two vectors. Given
            two vectors a = ⟨a1 , a2 , a3 ⟩ and b = ⟨b1 , b2 , b3 ⟩,

            a × b = (|a||b| sin θ)n = ⟨a2 b3 − a3 b2 , a3 b1 − a1 b3 , a1 b2 − a2 b1 ⟩,

            where n is a unit vector perpendicular to both a and b.
        ◮   To understand and be able to apply properties of the dot
            product, especially a × b = −b × a.
        ◮   To understand that two vectors are parallel if and only if their
            cross product is zero.
        ◮   To be able to use vectors and the cross product in applications.


                                                            .     .    .     .    .       .
Definition of the cross product
   Definition
   Given vectors a and b in space, the cross product of a and b is
   the vector
                        a × b = |a| |b| (sin θ) n,
   where n is a vector perpendicular to a and b such that (a, b, n) is a
   right-handed set of three vectors.
   In components, if

                        a = ⟨a1 , a2 , a3 ⟩= a1 i + a2 j + a3 k
                        b = ⟨b1 , b2 , b3 ⟩= b1 i + b2 j + b3 k

   Then

        a × b = (a2 b3 − b2 a3 )i + (a3 b1 − b3 a1 )j + (a1 b2 − b1 a2 )k
               = ⟨a 2 b 3 − b 2 a 3 , a 3 b 1 − b 3 a 1 , a 1 b 2 − b 1 a 2 ⟩

                                                                  .       .     .   .   .   .
Determinant formula



   This is only to help you remember, in case you’ve seen determinants
   of 3 × 3 matrices:

      i j k
                  a a     a a     a a
     a1 a2 a3 = i 2 3 − j 1 3 + k 1 2
                  b2 b3   b1 b 3  b1 b2
     b1 b2 b3
                 = (a2 b3 − b2 a3 )i − (b3 a1 − a3 b1 )j + (a1 b2 − b1 a2 )k
                 =a×b




                                                    .    .     .    .    .     .
Algebraic Properties of the Cross Product




   If a, b, and c are vectors and c is a scalar, then
   1. a × b = −b × a
   2. (ca) × b = c(a × b) = a × (cb)
   3. a × (b + c) = a × b + a × c
   4. (a + b) × c = a × c + b × c




                                                        .   .   .   .   .   .
Geometric Properties of the cross product



    ◮   a × b = 0 exactly when a and b are parallel.
    ◮   The magnitude of the cross product a × b is the area of the
        parallelogram with sides a and b.




                                  .
                                  b       |
                                          . b| sin θ

                                  θ
                                  .
                              .       .
                                      a




                                                       .   .   .   .   .   .
Applications of the cross product




     ◮   Area of a parallelogram
     ◮   Torque τ = r × F
     ◮   Volume of a parallelipiped (scalar triple product)

                                   V = |a · (b × c)|




                                                       .   .   .   .   .   .
The scalar triple product in action
   Example
   Find the volume of the parallelipiped spanned by a = ⟨2, 0, −3⟩,
   b = ⟨1, 1, 1⟩, and c = ⟨0, 4, −1⟩




                                                 .    .    .    .     .   .
The scalar triple product in action
      Example
      Find the volume of the parallelipiped spanned by a = ⟨2, 0, −3⟩,
      b = ⟨1, 1, 1⟩, and c = ⟨0, 4, −1⟩
      Solution


                                             The scalar triple product can be
                                             calculated by
        .
        b                                           2 0 −3
         .
         c                                          1 1 1 = −22
  .                                                 0 4 −1

                                             It’s negative because the triple
             .
             a                               (a, b, c) is left-handed. So the
                                             volume is 22.

                                                    .     .    .    .     .     .
Equations of Lines and Planes I



     ◮   To understand and be able to represent a line ℓ in R3 using
           ◮   the vector equation
                                          r = r0 + tv
           ◮   the parametric equations

                         x = x0 + at      y = y0 + bt       z = z0 + ct

           ◮   the symmetric equations
                                 x − x0   y − y0   z − z0
                                        =        =
                                   a        b         c




                                                        .     .    .      .   .   .
Equations of Lines and Planes II


     ◮   To be able to represent line segment in R3 from r0 to r1 ,

                               r(t) = (1 − t)r0 + tr1 ,

         where 0 ≤ t ≤ 1.
     ◮   To understand and be able to represent a plane in R3 given a
         normal vector n = ⟨a, b, c⟩ and a point P0 = (x0 , y0 , z0 ) in the
         plane,
                         n · ⟨x − x 0 , y − y 0 , z − z 0 ⟩ = 0
         or
                      a(x − x0 ) + b(y − y0 ) + c(z − z0 ) = 0.




                                                      .    .    .    .     .   .
Equations of Lines and Planes III




     ◮   To understand and be able to calculate the distance D from a
         point P1 = (x1 , y1 , z1 ) to the plane ax + by + cz + d = 0,

                                |ax1 + by1 + cz1 + d|
                           D=      √                  .
                                      a 2 + b2 + c2




                                                  .       .   .   .      .   .
Applying the definition
   Example
   Find the vector, parametric, and symmetric equations for the line
   that passes through (1, 2, 3) and (2, 3, 4).




                                                 .    .    .    .      .   .
Applying the definition
   Example
   Find the vector, parametric, and symmetric equations for the line
   that passes through (1, 2, 3) and (2, 3, 4).
   Solution
     ◮   Use the initial vector ⟨1, 2, 3⟩ and direction vector
         ⟨2, 3, 4⟩ − ⟨1, 2, 3⟩ = ⟨1, 1, 1⟩. Hence

                              r(t) = ⟨1, 2, 3⟩ + t ⟨1, 1, 1⟩

     ◮   The parametric equations are

                        x=1+t           y=2+t           z=3+t

     ◮   The symmetric equations are

                                x−1=y−2=z−3
                                                        .        .   .   .   .   .
Planes

   The set of all points whose displacement vector from a fixed point is
   perpendicular to a fixed vector is a plane.


                                         z
                                         .


                                 .
                                 n

                                         .0
                                         r
                                     .                    y
                                                          .


                          .
                          x



                                                 .    .       .   .   .   .
Equations for planes
   The plane passing through the point with position vector
   r0 = ⟨x0 , y0 , z0 ⟩ perpendicular to ⟨a, b, c⟩ has equations:
     ◮   The vector equation

                       n · (r − r0 ) = 0 ⇐⇒ n · r = n · r0

     ◮   Rewriting the dot product in component terms gives the scalar
         equation

                      a(x − x0 ) + b(y − y0 ) + c(z − z0 ) = 0

         The vector n is called a normal vector to the plane.
     ◮   Rearranging this gives the linear equation

                               ax + by + cz + d = 0,

         where d = −ax0 − by0 − cz0 .
                                                      .    .     .   .   .   .
Example
Find an equation of the plane that passes through the points
P(1, 2, 3), Q(3, 5, 7), and R(4, 3, 1).




                                              .    .    .      .   .   .
Example
Find an equation of the plane that passes through the points
P(1, 2, 3), Q(3, 5, 7), and R(4, 3, 1).

Solution−
        →                               −
                                        → −  →
Let r0 = OP = ⟨1, 2, 3⟩. To get n, take PQ × PR:

                         i j k
               −
               → −  →
               PQ × PR = 2 3 4 = ⟨−10, 16, −7⟩
                         3 1 −2

So the scalar equation is

               −10(x − 1) + 16(y − 2) − 7(z − 3) = 0.




                                                   .   .   .   .   .   .
Distance from point to line

   Definition
   The distance between a point and a line is the smallest distance
   from that point to all points on the line. You can find it by projection.

                                        Q
                                        .
                                            .

                                                         b·v
                                                .b −         v
                                                         v·v
                                  .
                                  b

                                                   b·v
                                      . rojv b =
                                      p                v
                              .
                              v                    v·v
                          θ
                          .
                      .
                     P
                     .0

                                                     .     .     .   .   .    .
Distance from a point to a plane
   Definition
   The distance between a point and a plane is the smallest distance
   from that point to all points on the line.
                                                     .
                                                         .
                                                         Q

                                          .
                                          b              |n · b|
                                                         .
                                                           |n|
                                  .
                                  n
                                     .
                                    P
                                    .0



   To find the distance from the a point to a plane, project the
   displacement vector from any point on the plane to the given point
   onto the normal vector.
                                                 .           .     .   .   .   .
Distance from a point to a plane II

   We have
                                         |n · b|
                                   D=
                                           |n|
   If Q = (x1 , y1 , z1 ), and the plane is given by ax + by + cz + d = 0, then
   n = ⟨a, b, c⟩, and

                n · b = ⟨a, b, c⟩ · ⟨x1 − x0 , y1 − y0 , z1 − z0 ⟩
                         = ax1 + by1 + cz1 − ax0 − by0 − cz0
                         = ax1 + by1 + cz1 + d

   So the distance between the plane ax + by + cz + d = 0 and the point
   (x1 , y1 , z1 ) is
                             |ax1 + by1 + cz1 + d|
                        D=      √
                                   a 2 + b 2 + c2


                                                        .     .      .   .   .    .
Functions and surfaces
Learning objectives for Section 9.6




        ◮   To understand and be able to represent a function of two
            variables, z = f(x, y), graphically. Section 9.6)
        ◮   To understand and be able to use the trace (cross-section) of a
            function. Section 9.6)
        ◮   To understand and be able to determine the domain of a
            function z = f(x, y). Section 9.6)
        ◮   To understand and be able to represent a selection of quadric
            surfaces graphically.




                                                     .    .    .    .   .     .
function, domain, range




   Definition
   A function f of two variables is a rule that assigns to each
   ordered pair of real numbers (x, y) in a set D a unique real number
   denoted by f(x, y). The set D is the domain of f and its range is
   the set of values that f takes on. That is

                      range f = { f(x, y) | (x, y) ∈ D }




                                                    .      .   .   .   .   .
Example


  Example
                                           √
  Find the domain and range of f(x, y) =       xy.




                                                     .   .   .   .   .   .
Example


  Example
                                               √
  Find the domain and range of f(x, y) =           xy.

  Solution
    ◮   Working from the outside in, we see that xy must be nonnegative,
        which means x ≥ 0 and y ≥ 0 or x ≤ 0 and y ≤ 0. Thus the domain
        is the union of the coordinate axes, and the first and third quadrants.
    ◮   The range of f is the set of all “outputs” of f. Clearly the range of f is
        restricted to the set of nonnegative numbers. To make sure that we
        can get all nonnegative numbers x, notice x = f(x, x).




                                                         .    .    .     .    .      .
Solution continued

   Here is a sketch of the domain:




                                     .




                                         .   .   .   .   .   .
Traces and surfaces



   Example
   Describe and sketch the surfaces.

     (i)   z = 4x2 + y2                 (vi) y2 − 9x2 − 4z2 = 36
    (ii)   z = 4 − x 2 − y2
                                       (vii) x2 + 4y2 + 2z2 = 4
   (iii)   z 2 = 4 (x 2 + y 2 )
   (iv)    x = 2y2 − z2                (viii) y2 + z2 = 1
    (v)    x2 + y2 − 9z2 = 9




                                                   .    .   .     .   .   .
Sketch of z = 4x2 + y2
   The trace in the xz-plane (y = 0) is the parabola z = 4x2 . The trace
   in the yz-plane is the parabola z = y2 . The trace in the xy-plane is the
   curve 4x2 + y2 = 0, which is just a point (0, 0). But the trace in the
   plane z = k (k > 0) is the ellipse 4x2 + y2 = k. So we get ellipses
   crossing the parabolas, an elliptic paraboloid.
                                                                   2


                                                         1


                                               0



                                    1



                         2
                      2.0


                       1.5


                       1.0


                        0.5


                         0.0
                             1.0
                                   0.5
                                         0.0
                                                   0.5
                                                             1.0

                                                                       .   .   .   .   .   .
Sketch of z2 = 4(x2 + y2 )
   The trace in the xz-plane is the equation z2 = 4x2 , or z = ±2x. The
   trace in the yz-plane is the equation z = ±2y. Traces in the plane
   z = k are circles k2 = 4(x2 + y2 ). So we have a cone.
                                                             1.0
                                                     0.5

                                               0.0

                                        0.5

                           1.0
                     1.0




                      0.5



                           0.0



                            0.5


                                 1.0
                                  1.0
                                              0.5
                                                       0.0

                                                                   0.5

                                                                         1.0




                                                                               .   .   .   .   .   .
Sketch of x = 2y2 − z2
   The trace in the xy-plane is a parabola x = 2y2 . The trace in the
   xz-plane is a parabola opening the other direction: x = −z2 . Traces
   x = k give hyperbolas. So we have a hyperbolic paraboloid.
                                                             1.0
                                                     0.5

                                               0.0

                                        0.5

                           1.0
                     1.0




                      0.5



                           0.0



                            0.5


                                 1.0
                                  1.0
                                              0.5
                                                       0.0

                                                                   0.5

                                                                         1.0




                                                                               .   .   .   .   .   .
Sketch of x2 + y2 − 9z2 = 9

   The x2 + y2 piece tells us that this is a surface of revolution, where
   the z-axis is the axis of revolution. If we find the xz-trace, we get
   x2 − 9z2 = 9, a hyperbola. We get a hyperboloid of one sheet.




                      2

                      1

                          0                                               4

                          1                                           2
                              2
                                  4                           0

                                      2

                                          0               2

                                              2
                                                      4
                                                  4




                                                                  .           .   .   .   .   .
Sketch of y2 − 9x2 − 4z2 = 36
   Traces in the xy and yz planes are hyperbolas. Traces in the plane
   y = k are ellipses 9x2 + 4z2 = k2 − 36. We have a hyperboloid of
   two sheets.
                                                   10



                                               5




                                           0




                                   5




                      10
                      4


                      2


                       0


                          2


                           4

                               2
                                   0
                                       2



                                                        .   .   .   .   .   .
Sketch of x2 + 4y2 + 2z2 = 4

   The traces in the planes x = 0, y = 0, and z = 0 are all ellipses. We
   get an ellipsoid.
                                              1.0 2
                                        0.5           1
                                  0.0                     0
                            0.5                               1
                      1.0
                                                                          2

                                                                                  2




                                                                              1




                                                                          0




                                                                          1



                                                                      2




                                                                  .                   .   .   .   .   .
Traces and surfaces


   Example
   Describe and sketch the surfaces.

     (i) z = 4x2 + y2 elliptic          (v) x2 + y2 − 9z2 = 9
         paraboloid                         hyperboloid of one sheet
    (ii) z = 4 − x2 − y2 elliptic       (vi) y2 − 9x2 − 4z2 = 36
         paraboloid                          hyperboloid of two sheets
   (iii) z2 = 4(x2 + y2 ) cone         (vii) x2 + 4y2 + 2z2 = 4 ellipsoid
   (iv) x = 2y2 − z2 hyperbolic        (viii) y2 + z2 = 1 cylinder
         paraboloid




                                                   .    .    .       .   .   .
Cylindrical and Spherical Coordinates I
Learning objectives for Section 9.7




        ◮   To understand and be able to apply the polar coordinate system in
            R2 , and to understand the relationship to rectangular
            coordinates:

                                      x = r cos θ   y = r sin θ
                                  2       2    2
                                 r =x +y            tan θ = y/x,

            where r ≥ 0 and 0 ≤ θ ≤ 2π .




                                                           .      .   .   .   .   .
Cylindrical and Spherical Coordinates II
Learning objectives for Section 9.7




        ◮   To understand and be able to apply the cylindrical coordinate
            system in R3 , and to understand the relationship to rectangular
            coordinates:

                             x = r cos θ   y = r sin θ       z=z
                          r 2 = x2 + y 2   tan θ = y/x       z = z,

            where r ≥ 0 and 0 ≤ θ ≤ 2π .




                                                         .    .    .   .   .   .
Cylindrical and Spherical Coordinates III
Learning objectives for Section 9.7




        ◮   To understand and be able to apply the spherical coordinate
            system in R3 , and to understand the relationship to rectangular
            coordinates:

                  x = ρ sin φ cos θ       y = ρ sin φ sin θ         z = ρ cos φ
                                      2      2    2     2
                                      ρ =x +y +z ,

            where r ≥ 0, 0 ≤ θ ≤ 2π , and 0 ≤ φ ≤ π .




                                                            .   .      .    .     .   .
Why different coordinate systems?




    ◮   The dimension of space comes from nature
    ◮   The measurement of space comes from us
    ◮   Different coordinate systems are different ways to measure
        space




                                                .    .    .   .      .   .
Vectors and the Geometry of Space

                             Conversion from polar to
                             cartesian (rectangular)

                                           x = r cos θ
                                           y = r sin θ

                             Conversion from cartesian to
              .
              r   .
                  y
            θ                polar:
          . .
              .
              x                          √
                                     r = x2 + y 2


                                       x               y                 y
                             cos θ =        sin θ =            tan θ =
                                       r               r                 x


                                       .      .    .       .      .      .
Cylindrical Coordinates
Just add the vertical dimension


                                                Conversion from cylindrical to
                                                cartesian (rectangular):
                 z
                 .
                                                  x = r cos θ        y = r sin θ
                               (
                               . r, θ, z)                      z=z

                                                Conversion from cartesian to
                             z z
                             . .                cylindrical:
                   .    r
                        .                   .
                                            y                √
                 θ
                 .                 y
                                   .
                                       .                 r = x2 + y 2
                       x
                       . .
                                                         x           y         y
     x
     .                                          cos θ =      sin θ =   tan θ =
                                                         r           r         x
                                                             z=z


                                                       .   .     .      .    .     .
Spherical Coordinates
like the earth, but not exactly

                                              Conversion from spherical to
                                              cartesian (rectangular):

                  z
                  .                                   x = ρ sin φ cos θ
                         .                            y = ρ sin φ sin θ
                             (
                             . r, θ, φ)               z = ρ cos φ
                         .
                                              Conversion from cartesian to
                    φ ρ
                    . .
                        z
                        .                     spherical:
                    .                     .
                                          y
                                                  √               √
             x
             .    θ
                  .
                                              r = x2 + y2 ρ = x2 + y2 + z2
                  y
                  .                                    x          y          y
      x
      .                                       cos θ =     sin θ =    tan θ =
                                                       r          r          x
                                                                  z
                                                         cos φ =
                                                                  ρ
                                                       .   .    .     .   .   .
Outline

   Vectors and the Geometry of           Derivatives and Integrals of
   Space                                 Vector Functions
      Three-Dimensional                  Arc Length (not Curvature)
      Coordinate Systems                 Motion in Space: Velocity and
      Vectors                            Acceleration
      The Dot Product                    Parametric Surfaces
      The Cross Product               Partial Derivatives
      Equations of Lines and Planes      Functions of Several Variables
      Functions and surfaces             Utility Functions and
      Cylindrical and Spherical          indifference curves
      Coordinates                        Limits and Continuity
   Vector Functions                      Partial Derivatives
      Vector Functions and Space         Tangent Planes and Linear
      Curves                             Approximations

                                                .    .    .    .   .      .
Vector Functions and Space Curves
Learning objectives for Section 10.1

        ◮   To understand and be able to apply the concept of a
            vector-valued function,

                      r(t) = ⟨f(t), g(t), h(t)⟩ = f(t)i + g(t)j + h(t)k.

        ◮   To be able to apply the concepts of limits and continuity to
            vector-valued functions.
        ◮   To understand that a curve in R3 can be represented
            parametrically

                              x = f(t)        y = g(t)       z = h (t)

            or by a vector-valued function

                                       r(t) = ⟨f(t), g(t), h(t)⟩

                                                              .    .     .   .   .   .
Example
Given the plane curve described by the vector equation

                      r(t) = sin(t)i + 2 cos(t)j


(a) Sketch the plane curve.




                                               .   .     .   .   .   .
Solution

                    r(t) = r(t) = sin(t)i + 2 cos(t)j




                                                            y
                                                            .


              t r (t)
              0 2j
            π/2   i                                 . (π/4)
                                                    r
             π −2j                                          .           x
                                                                        .
           3π/2 −i
            2π 2j



                                                .       .       .   .       .   .
Derivatives and Integrals of Vector Functions
Learning objectives for Section 10.2




        ◮   To understand and be able to calculate the derivative of a
            vector-valued function.
        ◮   To understand and be able to apply the basic rules of
            differentiation for vector-valued functions.
        ◮   To understand and be able to find the definite integral of a
            vector-valued function.




                                                      .    .    .   .    .   .
Derivatives of vector-valued functions


   Definition
   Let r be a vector function.
     ◮   The limit of r at a point a is defined componentwise:
                                 ⟨                           ⟩
                     lim r(t) = lim f(t), lim g(t), lim h(t)
                     t→a         t→a     t→a          t→a

     ◮   The derivative of r is defined in much the same way as it is
         for real-valued functions:
                        dr                r(t + h) − r(t)
                           = r′ (t) = lim
                        dt            h→0        h




                                                  .         .   .   .   .   .
Rules for differentiation

   Theorem
   Let u and v be differentiable vector functions, c a scalar, and f a
   real-valued function. Then:
         d
     1.     [u(t) + v(t)] = u′ (t) + v′ (t)
         dt
         d
     2.     [cu(t)] = cu′ (t)
         dt
         d
     3.     [f(t)u(t)] = f′ (t)u(t) + f(t)u′ (t)
         dt
         d
     4.     [u(t) · v(t)] = u′ (t) · v(t) + u(t) · v′ (t)
         dt
         d
     5.     [u(t) × v(t)] = u′ (t) × v(t) + u(t) × v′ (t)
         dt
         d
     6.     [u(f(t))] = f′ (t)u′ (f(t))
         dt

                                                       .    .    .       .   .   .
Example
Given the plane curve described by the vector equation

                      r(t) = sin(t)i + 2 cos(t)j


(a) Sketch the plane curve.
(b) Find r′ (t)




                                               .   .     .   .   .   .
Solution

                    r(t) = r(t) = sin(t)i + 2 cos(t)j
                    r′ (t) = cos(t)i − 2 sin(t)j


                                                               y
                                                               .


              t r (t)
              0 2j
            π/2   i                                    . (π/4)
                                                       r
             π −2j                                             .           x
                                                                           .
           3π/2 −i
            2π 2j



                                                   .       .       .   .       .   .
Example
Given the plane curve described by the vector equation

                      r(t) = sin(t)i + 2 cos(t)j


(a) Sketch the plane curve.
(b) Find r′ (t)
(c) Sketch the position vector r(π/4) and the tangent vector
    r′ (π/4).




                                               .   .     .   .   .   .
Solution

                    r(t) = r(t) = sin(t)i + 2 cos(t)j
                    r′ (t) = cos(t)i − 2 sin(t)j


                                                               y
                                                               .


              t r (t)
              0 2j
            π/2   i                                    . (π/4)
                                                       r               . ′ (π/4)
                                                                       r
             π −2j                                             .            .
                                                                            x
           3π/2 −i
            2π 2j



                                                   .       .       .    .       .   .
Integrals of vector-valued functions
   Definition
   Let r be a vector function defined on [a, b]. For each whole number
   n, divide the interval [a, b] into n pieces of equal width ∆t. Choose a
   point t∗ on each subinterval and form the Riemann sum
           i

                                       ∑
                                       n
                             Sn =             r(t∗ ) ∆t
                                                 i
                                        i=1

   Then define
   ∫ b                        ∑
                              n
       r(t) dt = lim Sn = lim   r(t∗ ) ∆t
                                   i
    a           n→∞         n→∞
                                      i=1
                                  [                                                                           ]
                                       ∑ n                    ∑
                                                              n                        ∑
                                                                                       n
                        = lim                f(t∗ ) ∆ti
                                                i         +          g(t∗ ) ∆tj
                                                                        i          +             h(t∗ ) ∆tk
                                                                                                    i
                            n→∞
                                       i=1                     i=1                         i=1
                            (∫    b           )      (∫       b            )       (∫        b        )
                        =             f(t) dt i +              g(t) dt j +                     h(t) dt k
                              a                           a                                a
                                                               .       .       .       .         .    .
FTC for vector functions

   Theorem (Second Fundamental Theorem of Calculus)
   If r(t) = R′ (t), then
                                    ∫       b
                                                r(t) dt = R(b) − R(a)
                                        a



   Example                                                 ∫       π
   Given r(t) = ⟨t, cos 2t, sin 2t⟩, find                               r(t) dt.
                                                               0

   Answer
             ⟨∫     π           ∫   π                 ∫    π                ⟩         ⟨               ⟩
                                                                                          π2
                        t dt,           cos 2t dt,             sin 2t dt          =          , 0, 0
                0               0                      0                                  2


                                                                                  .       .     .         .   .   .
Arc Length (not Curvature)
Learning objectives for Section 10.3




        ◮   To understand and be able to apply the arc length of a curve,
            r(t) = ⟨f(t), g(t), h(t)⟩, over an interval a ≤ t ≤ b,
                      ∫       b                  ∫ b√
                                    ′
                 L=               |r (t)| dt =          [f′ (t)]2 + [g′ (t)]2 + [h′ (t)]2 dt.
                        a                        a

        ◮   To understand and be able to apply the arc length function of a
            curve, r(t) = ⟨f(t), g(t), h(t)⟩, over an interval a ≤ t ≤ b,
                      ∫ t               ∫ t√
                            ′
              s (t) =     |r (u)| du =         [f′ (u)]2 + [g′ (u)]2 + [h′ (u)]2 du.
                          a                       a




                                                                      .     .     .     .       .   .
Length of a curve


   Break up the curve into pieces, and approximate the arc length with
   the sum of the lengths of the pieces:
        y
        .




        .                                                          .
                                                                   x


                                                 .   .    .    .       .   .
Length of a curve


   Break up the curve into pieces, and approximate the arc length with
   the sum of the lengths of the pieces:
        y
        .




        .                                                          .
                                                                   x


                                                 .   .    .    .       .   .
Length of a curve


   Break up the curve into pieces, and approximate the arc length with
   the sum of the lengths of the pieces:
        y
        .




        .                                                          .
                                                                   x


                                                 .   .    .    .       .   .
Length of a curve


   Break up the curve into pieces, and approximate the arc length with
   the sum of the lengths of the pieces:
        y
        .




        .                                                          .
                                                                   x


                                                 .   .    .    .       .   .
Length of a curve


   Break up the curve into pieces, and approximate the arc length with
   the sum of the lengths of the pieces:
        y
        .




        .                                                          .
                                                                   x


                                                 .   .    .    .       .   .
Length of a curve


   Break up the curve into pieces, and approximate the arc length with
   the sum of the lengths of the pieces:
        y
        .




                          ∑√
                          n
                    . ≈
                    L           (∆xi )2 + (∆yi )2
                          i=1




        .                                                           x
                                                                    .


                                                    .   .   .   .       .   .
Sum goes to integral
   If ⟨x, y⟩ is given by a vector-valued function r(t) = ⟨f(t), g(t), ⟩ with
   domain [a, b], we can approximate:
                        ∆xi ≈ f′ (ti )∆ti      ∆xi ≈ g′ (ti )∆ti
   So
             ∑√
             n                     ∑ √[
                                   n
                                                  ]2
        L≈     (∆xi )2 + (∆yi )2 ≈     f′ (ti )∆ti + [g′ (ti )∆ti ]2
              i=1                           i=1
                                           ∑ √[
                                           n
                                                       ]2
                                         =     f′ (ti ) + [g′ (ti )]2 ∆ti
                                            i=1
   As n → ∞, this converges to
                         ∫ b√
                     L=        [f′ (t)]2 + [g′ (t)]2 dt
                                  a
   In 3D, r(t) = ⟨f(t), g(t), h(t)⟩, and
                        ∫ b√
                   L=           [f′ (t)]2 + [g′ (t)]2 + [h′ (t)]2 dt
                            a
                                                           .    .      .   .   .   .
Example



  Example
  Find the length of the parabola y = x2 from x = 0 to x = 1.




                                                .    .    .     .   .   .
Example



  Example
  Find the length of the parabola y = x2 from x = 0 to x = 1.

  Solution ⟨      ⟩
  Let r(t) = t, t2 . Then
                    ∫   1√
                                           √
                                            5 1      √
               L=            1 + (2t)2 =     + ln 2 + 5
                    0                      2  4




                                                 .   .    .     .   .   .
Motion in Space: Velocity and Acceleration
Learning objectives for Section 10.4




        ◮   To understand the first derivative as the velocity of a curve and
            the second derivative as the acceleration of a curve and be able
            to apply these definitions.




                                                      .    .   .    .    .     .
Velocity and Acceleration




   Definition
   Let r(t) be a vector-valued function.
     ◮   The velocity v(t) is the derivative r′ (t)
     ◮   The speed is the length of the derivative |r′ (t)|
     ◮   The acceleration is the second derivative r′′ (t).




                                                      .   .   .   .   .   .
Example
Find the velocity, acceleration, and speed of a particle with position
function
                        r(t) = ⟨2 sin t, 5t, 2 cos t⟩




                                                .    .    .    .    .    .
Example
Find the velocity, acceleration, and speed of a particle with position
function
                        r(t) = ⟨2 sin t, 5t, 2 cos t⟩

Answer
  ◮   r′ (t) = ⟨2 cos(t), 5, −2 sin(t)⟩
                 √
  ◮    r′ (t) = 29
  ◮   r′′ (t) = ⟨−2 sin(t), 0, −2 cos(t)⟩




                                                .    .    .    .    .    .
Parametric Surfaces
Learning objectives for Section 10.5



        ◮   To understand and be able to apply the concept of a parametric
            surface. A parametric surface may be defined by a vector
            equation,
                           r(t) = x(u, v) i + y(u, v) j + z(u, v) k
            or by a set of parametric equations

                         x = x (u , v )   y = y(u, v)   z = z(u, v),

            where u and v are variables with a domain D contained in R2 .
        ◮   To understand and be able to represent surfaces of revolutions
            parametrically.



                                                        .    .    .    .   .   .
Surfaces with easy parametrizations




     ◮   graph
     ◮   plane
     ◮   sphere
     ◮   surface of revolution




                                      .   .   .   .   .   .
Parametrizing a graph



   Example
   Parametrize the surface described by

             z = x 2 + y2 ,   −2 ≤ x ≤ 2 ,   −3 ≤ y ≤ 3




                                             .   .   .    .   .   .
Parametrizing a graph



   Example
   Parametrize the surface described by

              z = x 2 + y2 ,   −2 ≤ x ≤ 2 ,      −3 ≤ y ≤ 3


   Solution
   Let
                     x = u,    y = v,      z = u 2 + v2
   on the domain −2 ≤ u ≤ 2, −3 ≤ v ≤ 3.




                                                 .    .   .   .   .   .
parametrizing a sphere



   Example
   Find a parametrization for the unit sphere.




                                                 .   .   .   .   .   .
parametrizing a sphere



   Example
   Find a parametrization for the unit sphere.

   Solution
   Use spherical coordinates. Let:

              x = cos θ sin φ,       y = sin θ sin φ,       z = cos φ

   The domain of parametrization is 0 ≤ θ ≤ 2π , 0 ≤ φ ≤ π .




                                                        .    .   .      .   .   .
Parametrizing a surface of revolution


   Example
   Find a parametrization of the surface described by

                    x2 − y 2 + z 2 = 1 ,   −3 ≤ y ≤ 3




                                                 .      .   .   .   .   .
Parametrizing a surface of revolution


   Example
   Find a parametrization of the surface described by

                     x2 − y 2 + z 2 = 1 ,     −3 ≤ y ≤ 3


   Solution
   The surface is the graph of z2 − y2 = 1 revolved around the y-axis. So let
                √                                     √
           x= 1+y        2 cos θ,     y = u,      z = 1 + y2 sin θ

   The domain is −3 ≤ u ≤ 3, 0 ≤ θ ≤ 2π .



                                                     .    .    .    .     .     .
Outline

   Vectors and the Geometry of           Derivatives and Integrals of
   Space                                 Vector Functions
      Three-Dimensional                  Arc Length (not Curvature)
      Coordinate Systems                 Motion in Space: Velocity and
      Vectors                            Acceleration
      The Dot Product                    Parametric Surfaces
      The Cross Product               Partial Derivatives
      Equations of Lines and Planes      Functions of Several Variables
      Functions and surfaces             Utility Functions and
      Cylindrical and Spherical          indifference curves
      Coordinates                        Limits and Continuity
   Vector Functions                      Partial Derivatives
      Vector Functions and Space         Tangent Planes and Linear
      Curves                             Approximations

                                                .    .    .    .   .      .
Functions of Several Variables
Learning objectives for Section 11.1




        ◮   To understand functions of several variables and be able to
            represent these functions using level sets.




                                                      .   .    .    .     .   .
A contour plot is a topographic map of a graph
                               √
   Consider the graph z = x2 + y2 Intersect the cone with planes
   z = c and what do you get? Circles. A contour plot shows evenly
   spaced circles.
   3



    2



    1



    0
                                       4
                                        3
   -1                                   2                                               2
                                         1
                                          0
   -2                                                                           0
                                              -2

                                                       0
                                                                           -2
   -3
        -3   -2   -1   0   1   2   3                           2




                                                   .       .       .   .            .       .
The paraboloid

   Example
   Graph z = x2 + y2 .
   3



    2



    1



    0

                                       15
                                       10
   -1                                                                                 2
                                        5
                                            0
                                                                              0
   -2                                           -2

                                                         0
                                                                         -2
   -3
        -3   -2   -1   0   1   2   3                             2




                                                     .       .   .   .            .       .
The hyperbolic paraboloid

   Example
   Graph z = x2 − y2 .
   3



    2



    1



    0

                                       5
   -1                                  0                                              2
                                       -5

   -2                                                                         0
                                            -2

                                                     0
                                                                         -2
   -3
        -3   -2   -1   0   1   2   3                         2




                                                 .       .       .   .            .       .
Plotting a Cobb-Douglas function

   Example
   Plot z = x1/2 y1/2 .

     3



   2.5



     2



   1.5                                     3

                                           2                                            3
     1                                         1
                                                                                    2
                                               0
                                               0
   0.5
                                                   1                            1

                                                               2
     0
         0   0.5   1   1.5   2   2.5   3                               30




                                                       .   .       .        .       .       .
Utility Functions and indifference curves




     ◮   If u is a utility function, a level curve of u is a curve along which
         all points have the same u value.
     ◮   We also know this as an indifference curve




                                                      .     .    .    .    .     .
Limits and Continuity
Learning objectives for Section 11.2




        ◮   To understand and be able to apply the concept of a limit of a
            function of several variables.
        ◮   To understand and be able to apply the definition of continuity
            for a function of several variables.




                                                      .   .    .    .    .   .
Definition
We write
                             lim        f(x, y) = L
                          (x,y)→(a,b)

and we say that the limit of f(x, y) as (x, y) approaches (a, b) is
L if we can make the values of f(x, y) as close to L as we like by taking
the point (x, y) to be sufficiently close to (a, b).




                                                      .   .   .   .   .     .
easy limits




     ◮      lim        x=a
         (x,y)→(a,b)
     ◮      lim        y=b
         (x,y)→(a,b)
     ◮      lim        c=c
         (x,y)→(a,b)




                             .   .   .   .   .   .
Like regular limits, limits of multivariable functions can be
  ◮   added
  ◮   subtracted
  ◮   multiplied
  ◮   composed
  ◮   divided, provided the limit of the denominator is not zero.




                                                 .     .    .   .   .   .
Limit of a Polynomial


   Example
   Find       lim        (x5 + 4x3 y − 5xy2 )
          (x,y)→(5,−2)




                                                .   .   .   .   .   .
Limit of a Polynomial


   Example
   Find       lim        (x5 + 4x3 y − 5xy2 )
          (x,y)→(5,−2)

   Solution

            lim     (x5 + 4x3 y − 5xy2 ) = (5)5 + 4(5)3 (−2) − 5(5)(−2)2
     (x,y)→(5,−2)

                                           = 3125 + 4(125)(−2) − 5(5)(4)
                                           = 2025.




                                                       .   .   .   .       .   .
Limit of a Rational Expression


   Example
   Compute
                                      x2
                          lim              .
                       (x,y)→(1,2) x2 + y2




                                               .   .   .   .   .   .
Limit of a Rational Expression


   Example
   Compute
                                         x2
                             lim              .
                          (x,y)→(1,2) x2 + y2



   Solution

                                x2        (1)2
                    lim              =
                 (x,y)→(1,2) x2 + y2   (1)2 + (2)2
                                       1
                                     =
                                       5



                                                  .   .   .   .   .   .
What can go wrong?




  The only real problem is a limit where the denominator goes to zero.

    ◮   If the numerator goes to some number and the denominator
        goes to zero then the quotient cannot have a limit.




                                               .    .    .   .    .      .
What can go wrong?




  The only real problem is a limit where the denominator goes to zero.

    ◮   If the numerator goes to some number and the denominator
        goes to zero then the quotient cannot have a limit.
    ◮   If on the other hand the numerator and denominator both go to
        zero we have no clue.




                                               .    .    .   .    .      .
Showing a limit doesn’t exist



   Theorem
   Suppose      lim        f(x, y) = L. Then the limit of f as (x, y) → (a, b) is L
             (x,y)→(a,b)
   along all paths through (a, b).
   There are two contrapositives to this statement:
     ◮   If there is a path through (a, b) along which the limit does not exist,
         the two-dimensional limit does not exist
     ◮   If there are two paths through (a, b) along which the limits exist but
         disagree, the two-dimensional limit does not exist




                                                           .     .    .    .     .    .
Continuity




   Definition
   A function f of two variables is called continuous at (a, b) if

                            lim        f(x, y) = f(a, b).
                         (x,y)→(a,b)

   We say f is continuous on D if f is continuous at every point (a, b)
   in D.




                                                        .   .   .   .   .   .
Partial Derivatives
Learning objectives for Section 11.3




        ◮   To understand and be able to apply the definition of a partial
            derivative.
        ◮   To be able to compute partial derivatives.
        ◮   To understand and be able to apply Clairaut’s Theorem. If f is
            defined on a disk D that contains the point (a, b) and the
            functions fxy and fyx are continuous on D, then

                                       fxy (a, b) = fyx (a, b).

        ◮   To understand the idea of a partial differential equation, and to
            be able to verify solutions to partial differential equations.



                                                                  .   .   .   .   .   .
Definition
Let f(x, y) be a function of two variables. We define the partial
                 ∂f      ∂f
derivatives         and     at a point (a, b) as
                ∂x       ∂y

                 ∂f              f (a + h, b) − f (a, b)
                    (a, b) = lim
                 ∂x          h→0            h
                 ∂f              f (a, b + h) − f (a, b)
                    (a, b) = lim
                 ∂y          h→0            h

In other words, we temporarily treat the other variable as constant
and differentiate the resulting one-variable function as in Calculus I.




                                                 .    .    .    .    .    .
Example
Let f(x, y) = x3 − 3xy2 . Find its partial derivatives.




                                                   .      .   .   .   .   .
Example
Let f(x, y) = x3 − 3xy2 . Find its partial derivatives.

Solution
              ∂f
When finding      , we hold y constant. So
              ∂x
                  ∂f               ∂
                     = 3x2 − (3y2 ) (x) = 3x2 − 3y2
                  ∂x               ∂x
Similarly,
                        ∂f
                           = 0 − 3x(2y) = −6xy
                        ∂y




                                                   .      .   .   .   .   .
Second derivatives

   If f(x, y) is a function of two variables, each of its partial derivatives
   are function of two variables, and we can hope that they are
   differentiable, too. So we define the second partial derivatives.
                                          ( )
                                ∂2f    ∂ ∂f
                                     =            = fxx
                                ∂ x2   ∂x ∂x
                                          ( )
                               ∂2f     ∂ ∂f
                                     =            = fxy
                             ∂y ∂x     ∂y ∂x
                                          ( )
                               ∂2f     ∂ ∂f
                                     =            = fyx
                             ∂x ∂y     ∂x ∂y
                                          ( )
                                ∂2f    ∂ ∂f
                                     =           = fyy
                                ∂ y2   ∂y ∂y




                                                    .     .    .    .    .      .
Don’t worry about the mixed partials



   The “mixed partials” bookkeeping may seem scary. However, we are
   saved by:
   Theorem (Clairaut’s Theorem/Young’s Theorem)
   If f is defined near (a, b) and fxy and fyx are continuous at (a, b), then

                               fxy (a, b) = fyx (a, b).

   The upshot is that we needn’t worry about the ordering.




                                                          .   .   .    .       .   .
Example (Continued)
Let f(x, y) = x3 − 3xy2 . Find the second derivatives of f.




                                                 .    .       .   .   .   .
Example (Continued)
Let f(x, y) = x3 − 3xy2 . Find the second derivatives of f.

Solution
We have

                       fxx = (3x2 − 3y2 )x = 6x
                       fxy = (3x2 − 3y2 )y = −6y
                       fyx = (−6xy)x = −6y
                       fyy = (−6xy)y = −6x

Notice that fyx = fxy , as predicted by Clairaut (everything is a
polynomial here so there are no concerns about continuity). The fact
that fxx = fyy is a coincidence.


                                                  .   .       .   .   .   .
Partial Differential Equations

   Definition
   A partial differential equation (PDE) is a differential equation
   for a function of more than one variable. So the derivatives involved
   are partial derivatives.




                                                 .    .    .   .    .      .
Partial Differential Equations

   Definition
   A partial differential equation (PDE) is a differential equation
   for a function of more than one variable. So the derivatives involved
   are partial derivatives.

   Example
   Let f(x, y) = x3 − 3xy2 . Show that f satisfies the Laplace equation

                              ∂2f  ∂2f
                                  + 2 =0
                              ∂ x2 ∂ y




                                                 .    .    .   .    .      .
Partial Differential Equations

   Definition
   A partial differential equation (PDE) is a differential equation
   for a function of more than one variable. So the derivatives involved
   are partial derivatives.

   Example
   Let f(x, y) = x3 − 3xy2 . Show that f satisfies the Laplace equation

                              ∂2f  ∂2f
                                  + 2 =0
                              ∂ x2 ∂ y


   Solution
   We already showed that fxx = 6x and fyy = −6x. So fxx + fyy = 0.


                                                  .    .    .    .    .    .
Tangent Planes and Linear Approximations I
Learning objectives for Section 11.4




        ◮   To understand the concept of a tangent plane to a surface
            z = f(x, y) and to be able to compute the equation of tangent
            planes
        ◮   To understand and be able to find a linear approximation to a
            function z = f(x, y).




                                                     .    .    .   .    .   .
Tangent Planes and Linear Approximations II
Learning objectives for Section 11.4




        ◮   To understand the concept of a tangent plane to a parametrically
            defined surface

                           r(u, v) = x(u, v) i + y(u, v) j + z(u, v) k

            and to be able to compute the equation of tangent planes.




                                                           .    .    .   .   .   .
Tangent Planes and Linear Approximations III
Learning objectives for Section 11.4




        ◮   To understand and be able to find the differential to a function
            z = f(x, y),
                                       ∂z      ∂z
                                 dz =     dx + dy
                                       ∂x      ∂y
            To be able to use the differential to estimate maximum error.




                                                      .    .    .   .    .    .
Definition
If f is a function and P0 = (x0 , y0 , z0 = f(x0 , y0 ) a point on its graph,
then
  ◮   The linearization of f near (x0 , y0 ) is the function

                                  ∂f                      ∂f
        L(x, y) = f(x0 , y0 ) +      (x0 , y0 )(x − x0 ) + (x0 , y0 )(y − y0 )
                                  ∂x                      ∂y
  ◮   The tangent plane to the graph at P0 is the graph of L

The linearization is the best possible linear approximation to a
function at a point.




                                                       .    .     .    .     .   .
Example
Find an equation of the tangent plane to the surface z = y cos(x − y)
at (2, 2, 2).




                                               .   .    .    .    .     .
Example
Find an equation of the tangent plane to the surface z = y cos(x − y)
at (2, 2, 2).

Solution
We have
       ∂z                            ∂z
          = −y sin(x − y),              = cos(x − y) + y sin(x − y)
       ∂x                            ∂y

So
                     ∂z                   ∂z
                                  = 0,                 =1
                     ∂x   (2,2)           ∂y   (2,2)

Therefore the equation of the tangent plane is

                   z = 2 + 0(x − 2) + 1(y − 2) = y


                                                       .    .   .   .   .   .
Example
The number of units of output per day at a factory is
                             [                 ]−1/2
                              1 −2  9
               P(x, y) = 150    x + y−2                 ,
                             10    10

where x denotes capital investment (in units of $1000), and y denotes
the total number of hours (in units of 10) the work force is employed
per day. Suppose that currently, capital investment is $50,000 and the
total number of working hours per day is 500. Estimate the change in
output if capital investment is increased by $5000 and the number of
working hours is decreased by 10 per day.




                                               .    .       .   .   .    .
Solution
                           ( )[               ]    ( )
           ∂P                   1 1 −2    9 −2 −3/2 −2 −3
              (x, y) = 150 −         x + y             x
           ∂x                   2 10     10         10
                          [            ]
                             1 −2  9 −2 −3/2 −3
                     = 15      x + y        x
                            10    10
       ∂P
          (50, 50) = 15
       ∂x

                        ( )[                ]− 3 / 2 ( )
        ∂P                   1  1 −2    9             9
           (x, y) = 150 −         x + y−2                (−2)y−3
        ∂y                   2 10      10             10
                       [             ]
                         1 −2   9 −2 −3/2 −3
                  = 15      x + y         x
                         10    10
     ∂P
        (50, 50) = 135
     ∂y
   So the linear approximation is L = 7500 + 15(x − 50) + 135(y − 50)
                                               .    .   .    .    .     .
Solution, continued


   So the linear approximation is

                   L = 7500 + 15(x − 50) + 135(y − 50)

   If ∆x = 5 and ∆y = −1, then

                  L = 7500 + 15 · 5 + 135 · (−1) = 7440

   The actual value is
                             P(55, 49) ≈ 7427
                       13
   So we are off by        ≈ 1.75%
                      7427



                                                .   .     .   .   .   .
Tangent planes to parametrized surfaces
    ◮   If r(u, v) parametrizes a surface S, and P0 = r(u0 , v0 ), then the
        vector
                                         r(u0 + h, v0 ) − r(u0 , v0 )
                     ru (u0 , v0 ) = lim
                                     h→0             h
        is tangent to S at P.
    ◮   If r(u, v) = x(u, v)i + y(u, v)j + z(u, v)k, then

                                ∂x              ∂y            ∂z
              ru (u0 , v0 ) =      (u0 , v0 )i + (u0 , v0 )j + (u0 , v0 )k
                                ∂u              ∂u            ∂u
    ◮   We have another tangent vector

                                ∂x              ∂y            ∂z
              rv (u0 , v0 ) =      (u0 , v0 )i + (u0 , v0 )j + (u0 , v0 )k
                                ∂v              ∂v            ∂v
    ◮   Their cross product is normal to the tangent plane.
                                                        .     .    .    .    .   .
Example
Find an equation of the tangent plane to the parametric surface

                 r(u, v) = u2 i + 2u sin v j + u cos v k

at the point u = 1 and v = 0.




                                                 .     .   .   .   .   .
Example
Find an equation of the tangent plane to the parametric surface

                  r(u, v) = u2 i + 2u sin v j + u cos v k

at the point u = 1 and v = 0.
Solution
The point we are finding the tangent plane at is r(1, 0) = ⟨1, 0, 1⟩. The
tangent plane is spanned by the two vectors ru (1, 0) and rv (1, 0):

    ru (u, v) = ⟨2u, 2 sin v, cos v⟩         =⇒ ru (1, 0) = ⟨2, 0, 1⟩
     rv (u, v) = ⟨0, 2u cos v, −u sin v⟩     =⇒ rv (1, 0) = ⟨0, 2, 0⟩

So a normal vector to the tangent plane is

        ru (1, 0) × rv (1, 0) = ⟨2, 0, 1⟩ × ⟨0, 2, 0⟩ = ⟨−2, 0, 4⟩

This means an equation for the tangent plane is

                       −2(x − 1) + 4(z − 1) = 0.   .    .   .    .      .   .
Differentials


   This is really just another way to express linear approximation.
   Define new variables dx = ∆x, dy = ∆y, dz. Then the equation for
   the tangent plane through (x0 , y0 , z0 = f(x0 , y0 )) is

                                  ∂f      ∂f
                           dz =      dx +    dy
                                  ∂x      ∂y

   And the concept that this is a good linear approximation is expressed
   as
                  ∆z = f(x + dx, y + dy) − f(x, y) ≈ dx
   when dx and dy are “small enough”




                                                  .   .    .   .    .      .
Example
If z = 5x2 + y2 and (x, y) changes from (1, 2) to (1.05, 2.1), compare
the values of ∆z and dz.




                                               .    .    .    .    .     .
Example
If z = 5x2 + y2 and (x, y) changes from (1, 2) to (1.05, 2.1), compare
the values of ∆z and dz.

Solution
We have
                         dz = 10x dx + 2y dy
When x = 1, y = 2, dx = 0.05, and dy = 0.1, we get dz = 0.9. On the
other hand
                       z(1.05, 2.1) = 9.9225
So ∆z = 0.9225. The difference is 0.0225, or 2.4%.




                                               .     .   .    .    .     .

Contenu connexe

Tendances

Tendances (20)

Pc12 sol c03_3-5
Pc12 sol c03_3-5Pc12 sol c03_3-5
Pc12 sol c03_3-5
 
Differential equation
Differential equation Differential equation
Differential equation
 
Unit iii
Unit iiiUnit iii
Unit iii
 
Lesson 11: Implicit Differentiation
Lesson 11: Implicit DifferentiationLesson 11: Implicit Differentiation
Lesson 11: Implicit Differentiation
 
001 basic concepts
001 basic concepts001 basic concepts
001 basic concepts
 
Chapter 4(differentiation)
Chapter 4(differentiation)Chapter 4(differentiation)
Chapter 4(differentiation)
 
Derivadas
DerivadasDerivadas
Derivadas
 
Actividad 4 calculo diferencial
Actividad 4 calculo diferencialActividad 4 calculo diferencial
Actividad 4 calculo diferencial
 
Algo Final
Algo FinalAlgo Final
Algo Final
 
Em10 fl
Em10 flEm10 fl
Em10 fl
 
Lesson 25: Unconstrained Optimization I
Lesson 25: Unconstrained Optimization ILesson 25: Unconstrained Optimization I
Lesson 25: Unconstrained Optimization I
 
09 trial melaka_s2
09 trial melaka_s209 trial melaka_s2
09 trial melaka_s2
 
Day 01
Day 01Day 01
Day 01
 
Maths model%20 qp
Maths model%20 qpMaths model%20 qp
Maths model%20 qp
 
Ma 104 differential equations
Ma 104 differential equationsMa 104 differential equations
Ma 104 differential equations
 
09 sistema de equação do primeiro grau
09 sistema de equação do primeiro grau09 sistema de equação do primeiro grau
09 sistema de equação do primeiro grau
 
Xi maths ch3_trigo_func_formulae
Xi maths ch3_trigo_func_formulaeXi maths ch3_trigo_func_formulae
Xi maths ch3_trigo_func_formulae
 
Chapter 07
Chapter 07Chapter 07
Chapter 07
 
Chapter 15
Chapter 15Chapter 15
Chapter 15
 
VIT - Mathematics -2007 Unsolved Paper
VIT - Mathematics -2007 Unsolved PaperVIT - Mathematics -2007 Unsolved Paper
VIT - Mathematics -2007 Unsolved Paper
 

En vedette

Global Warming Economic Impacts on Tanzania and Deforestation
Global Warming Economic Impacts on Tanzania and DeforestationGlobal Warming Economic Impacts on Tanzania and Deforestation
Global Warming Economic Impacts on Tanzania and DeforestationZ3P
 
Farming systems analysis: Tanzania, Malawi and Zambia
Farming systems analysis: Tanzania, Malawi and ZambiaFarming systems analysis: Tanzania, Malawi and Zambia
Farming systems analysis: Tanzania, Malawi and Zambiaafrica-rising
 
Spatial and temporal patterns of Rift Valley fever outbreaks in Tanzania: 193...
Spatial and temporal patterns of Rift Valley fever outbreaks in Tanzania: 193...Spatial and temporal patterns of Rift Valley fever outbreaks in Tanzania: 193...
Spatial and temporal patterns of Rift Valley fever outbreaks in Tanzania: 193...Global Risk Forum GRFDavos
 
Solution Manual : Chapter - 06 Application of the Definite Integral in Geomet...
Solution Manual : Chapter - 06 Application of the Definite Integral in Geomet...Solution Manual : Chapter - 06 Application of the Definite Integral in Geomet...
Solution Manual : Chapter - 06 Application of the Definite Integral in Geomet...Hareem Aslam
 
Lesson 8: Curves, Arc Length, Acceleration
Lesson 8: Curves, Arc Length, AccelerationLesson 8: Curves, Arc Length, Acceleration
Lesson 8: Curves, Arc Length, AccelerationMatthew Leingang
 
Lesson 20: (More) Optimization Problems
Lesson 20: (More) Optimization ProblemsLesson 20: (More) Optimization Problems
Lesson 20: (More) Optimization ProblemsMatthew Leingang
 
Lesson 17: The Mean Value Theorem and the shape of curves
Lesson 17: The Mean Value Theorem and the shape of curvesLesson 17: The Mean Value Theorem and the shape of curves
Lesson 17: The Mean Value Theorem and the shape of curvesMatthew Leingang
 
Lesson 19: Optimization Problems
Lesson 19: Optimization ProblemsLesson 19: Optimization Problems
Lesson 19: Optimization ProblemsMatthew Leingang
 
Lesson 9: Parametric Surfaces
Lesson 9: Parametric SurfacesLesson 9: Parametric Surfaces
Lesson 9: Parametric SurfacesMatthew Leingang
 
Lesson 9: The Product and Quotient Rule
Lesson 9: The Product and Quotient RuleLesson 9: The Product and Quotient Rule
Lesson 9: The Product and Quotient RuleMatthew Leingang
 
Lesson 19: Double Integrals over General Regions
Lesson 19: Double Integrals over General RegionsLesson 19: Double Integrals over General Regions
Lesson 19: Double Integrals over General RegionsMatthew Leingang
 
Lesson 10: Functions and Level Sets
Lesson 10: Functions and Level SetsLesson 10: Functions and Level Sets
Lesson 10: Functions and Level SetsMatthew Leingang
 
Lesson18 Double Integrals Over Rectangles Slides
Lesson18   Double Integrals Over Rectangles SlidesLesson18   Double Integrals Over Rectangles Slides
Lesson18 Double Integrals Over Rectangles SlidesMatthew Leingang
 
Lesson 8: Derivatives of Polynomials and Exponential functions
Lesson 8: Derivatives of Polynomials and Exponential functionsLesson 8: Derivatives of Polynomials and Exponential functions
Lesson 8: Derivatives of Polynomials and Exponential functionsMatthew Leingang
 
Lesson 10: Derivatives of Trigonometric Functions
Lesson 10: Derivatives of Trigonometric FunctionsLesson 10: Derivatives of Trigonometric Functions
Lesson 10: Derivatives of Trigonometric FunctionsMatthew Leingang
 
Lesson 7: Vector-valued functions
Lesson 7: Vector-valued functionsLesson 7: Vector-valued functions
Lesson 7: Vector-valued functionsMatthew Leingang
 
Lesson 17: The Method of Lagrange Multipliers
Lesson 17: The Method of Lagrange MultipliersLesson 17: The Method of Lagrange Multipliers
Lesson 17: The Method of Lagrange MultipliersMatthew Leingang
 
Lesson 7: What does f' say about f?
Lesson 7: What does f' say about f?Lesson 7: What does f' say about f?
Lesson 7: What does f' say about f?Matthew Leingang
 
Lesson 20: Integration in Polar Coordinates
Lesson 20: Integration in Polar CoordinatesLesson 20: Integration in Polar Coordinates
Lesson 20: Integration in Polar CoordinatesMatthew Leingang
 

En vedette (20)

Global Warming Economic Impacts on Tanzania and Deforestation
Global Warming Economic Impacts on Tanzania and DeforestationGlobal Warming Economic Impacts on Tanzania and Deforestation
Global Warming Economic Impacts on Tanzania and Deforestation
 
Farming systems analysis: Tanzania, Malawi and Zambia
Farming systems analysis: Tanzania, Malawi and ZambiaFarming systems analysis: Tanzania, Malawi and Zambia
Farming systems analysis: Tanzania, Malawi and Zambia
 
Spatial and temporal patterns of Rift Valley fever outbreaks in Tanzania: 193...
Spatial and temporal patterns of Rift Valley fever outbreaks in Tanzania: 193...Spatial and temporal patterns of Rift Valley fever outbreaks in Tanzania: 193...
Spatial and temporal patterns of Rift Valley fever outbreaks in Tanzania: 193...
 
Solution Manual : Chapter - 06 Application of the Definite Integral in Geomet...
Solution Manual : Chapter - 06 Application of the Definite Integral in Geomet...Solution Manual : Chapter - 06 Application of the Definite Integral in Geomet...
Solution Manual : Chapter - 06 Application of the Definite Integral in Geomet...
 
Lesson 8: Curves, Arc Length, Acceleration
Lesson 8: Curves, Arc Length, AccelerationLesson 8: Curves, Arc Length, Acceleration
Lesson 8: Curves, Arc Length, Acceleration
 
Lesson 20: (More) Optimization Problems
Lesson 20: (More) Optimization ProblemsLesson 20: (More) Optimization Problems
Lesson 20: (More) Optimization Problems
 
Lesson 18: Graphing
Lesson 18: GraphingLesson 18: Graphing
Lesson 18: Graphing
 
Lesson 17: The Mean Value Theorem and the shape of curves
Lesson 17: The Mean Value Theorem and the shape of curvesLesson 17: The Mean Value Theorem and the shape of curves
Lesson 17: The Mean Value Theorem and the shape of curves
 
Lesson 19: Optimization Problems
Lesson 19: Optimization ProblemsLesson 19: Optimization Problems
Lesson 19: Optimization Problems
 
Lesson 9: Parametric Surfaces
Lesson 9: Parametric SurfacesLesson 9: Parametric Surfaces
Lesson 9: Parametric Surfaces
 
Lesson 9: The Product and Quotient Rule
Lesson 9: The Product and Quotient RuleLesson 9: The Product and Quotient Rule
Lesson 9: The Product and Quotient Rule
 
Lesson 19: Double Integrals over General Regions
Lesson 19: Double Integrals over General RegionsLesson 19: Double Integrals over General Regions
Lesson 19: Double Integrals over General Regions
 
Lesson 10: Functions and Level Sets
Lesson 10: Functions and Level SetsLesson 10: Functions and Level Sets
Lesson 10: Functions and Level Sets
 
Lesson18 Double Integrals Over Rectangles Slides
Lesson18   Double Integrals Over Rectangles SlidesLesson18   Double Integrals Over Rectangles Slides
Lesson18 Double Integrals Over Rectangles Slides
 
Lesson 8: Derivatives of Polynomials and Exponential functions
Lesson 8: Derivatives of Polynomials and Exponential functionsLesson 8: Derivatives of Polynomials and Exponential functions
Lesson 8: Derivatives of Polynomials and Exponential functions
 
Lesson 10: Derivatives of Trigonometric Functions
Lesson 10: Derivatives of Trigonometric FunctionsLesson 10: Derivatives of Trigonometric Functions
Lesson 10: Derivatives of Trigonometric Functions
 
Lesson 7: Vector-valued functions
Lesson 7: Vector-valued functionsLesson 7: Vector-valued functions
Lesson 7: Vector-valued functions
 
Lesson 17: The Method of Lagrange Multipliers
Lesson 17: The Method of Lagrange MultipliersLesson 17: The Method of Lagrange Multipliers
Lesson 17: The Method of Lagrange Multipliers
 
Lesson 7: What does f' say about f?
Lesson 7: What does f' say about f?Lesson 7: What does f' say about f?
Lesson 7: What does f' say about f?
 
Lesson 20: Integration in Polar Coordinates
Lesson 20: Integration in Polar CoordinatesLesson 20: Integration in Polar Coordinates
Lesson 20: Integration in Polar Coordinates
 

Similaire à Math 21a Midterm I Review

Lesson 1: Functions and their Representations
Lesson 1: Functions and their RepresentationsLesson 1: Functions and their Representations
Lesson 1: Functions and their RepresentationsMatthew Leingang
 
Lesson03 The Concept Of Limit 027 Slides
Lesson03   The Concept Of Limit 027 SlidesLesson03   The Concept Of Limit 027 Slides
Lesson03 The Concept Of Limit 027 SlidesMatthew Leingang
 
Multiple integrals
Multiple integralsMultiple integrals
Multiple integralsTarun Gehlot
 
Multiple integrals
Multiple integralsMultiple integrals
Multiple integralsTarun Gehlot
 
Lesson 24: Optimization II
Lesson 24: Optimization IILesson 24: Optimization II
Lesson 24: Optimization IIMatthew Leingang
 
numarial analysis presentation
numarial analysis presentationnumarial analysis presentation
numarial analysis presentationOsama Tahir
 
Lesson 22: Optimization II (Section 4 version)
Lesson 22: Optimization II (Section 4 version)Lesson 22: Optimization II (Section 4 version)
Lesson 22: Optimization II (Section 4 version)Matthew Leingang
 
Lesson 22: Optimization II (Section 10 version)
Lesson 22: Optimization II (Section 10 version)Lesson 22: Optimization II (Section 10 version)
Lesson 22: Optimization II (Section 10 version)Matthew Leingang
 
Ai32647651
Ai32647651Ai32647651
Ai32647651IJMER
 
Lesson 5: Functions and surfaces
Lesson 5: Functions and surfacesLesson 5: Functions and surfaces
Lesson 5: Functions and surfacesMatthew Leingang
 
X2 t07 03 addition, subtraction, multiplication & division (2012)
X2 t07 03 addition, subtraction,  multiplication & division (2012)X2 t07 03 addition, subtraction,  multiplication & division (2012)
X2 t07 03 addition, subtraction, multiplication & division (2012)Nigel Simmons
 
X2 T07 03 addition, subtraction, multiplication & division (2011)
X2 T07 03 addition, subtraction,  multiplication & division (2011)X2 T07 03 addition, subtraction,  multiplication & division (2011)
X2 T07 03 addition, subtraction, multiplication & division (2011)Nigel Simmons
 
X2 T04 03 cuve sketching - addition, subtraction, multiplication and division
X2 T04 03 cuve sketching - addition, subtraction,  multiplication and divisionX2 T04 03 cuve sketching - addition, subtraction,  multiplication and division
X2 T04 03 cuve sketching - addition, subtraction, multiplication and divisionNigel Simmons
 
Exercise set 3.5
Exercise set 3.5Exercise set 3.5
Exercise set 3.5math265
 
short course on Subsurface stochastic modelling and geostatistics
short course on Subsurface stochastic modelling and geostatisticsshort course on Subsurface stochastic modelling and geostatistics
short course on Subsurface stochastic modelling and geostatisticsAmro Elfeki
 
Integration worksheet.
Integration worksheet.Integration worksheet.
Integration worksheet.skruti
 
Midterm Study Guide
Midterm Study GuideMidterm Study Guide
Midterm Study Guidevhiggins1
 

Similaire à Math 21a Midterm I Review (20)

Lesson 1: Functions
Lesson 1: FunctionsLesson 1: Functions
Lesson 1: Functions
 
Lesson 1: Functions and their Representations
Lesson 1: Functions and their RepresentationsLesson 1: Functions and their Representations
Lesson 1: Functions and their Representations
 
Lesson03 The Concept Of Limit 027 Slides
Lesson03   The Concept Of Limit 027 SlidesLesson03   The Concept Of Limit 027 Slides
Lesson03 The Concept Of Limit 027 Slides
 
Multiple integrals
Multiple integralsMultiple integrals
Multiple integrals
 
Multiple integrals
Multiple integralsMultiple integrals
Multiple integrals
 
Lesson 24: Optimization II
Lesson 24: Optimization IILesson 24: Optimization II
Lesson 24: Optimization II
 
numarial analysis presentation
numarial analysis presentationnumarial analysis presentation
numarial analysis presentation
 
Lesson 22: Optimization II (Section 4 version)
Lesson 22: Optimization II (Section 4 version)Lesson 22: Optimization II (Section 4 version)
Lesson 22: Optimization II (Section 4 version)
 
Lesson 22: Optimization II (Section 10 version)
Lesson 22: Optimization II (Section 10 version)Lesson 22: Optimization II (Section 10 version)
Lesson 22: Optimization II (Section 10 version)
 
Ai32647651
Ai32647651Ai32647651
Ai32647651
 
Lesson 5: Functions and surfaces
Lesson 5: Functions and surfacesLesson 5: Functions and surfaces
Lesson 5: Functions and surfaces
 
Notes
NotesNotes
Notes
 
X2 t07 03 addition, subtraction, multiplication & division (2012)
X2 t07 03 addition, subtraction,  multiplication & division (2012)X2 t07 03 addition, subtraction,  multiplication & division (2012)
X2 t07 03 addition, subtraction, multiplication & division (2012)
 
Lecture 1
Lecture 1Lecture 1
Lecture 1
 
X2 T07 03 addition, subtraction, multiplication & division (2011)
X2 T07 03 addition, subtraction,  multiplication & division (2011)X2 T07 03 addition, subtraction,  multiplication & division (2011)
X2 T07 03 addition, subtraction, multiplication & division (2011)
 
X2 T04 03 cuve sketching - addition, subtraction, multiplication and division
X2 T04 03 cuve sketching - addition, subtraction,  multiplication and divisionX2 T04 03 cuve sketching - addition, subtraction,  multiplication and division
X2 T04 03 cuve sketching - addition, subtraction, multiplication and division
 
Exercise set 3.5
Exercise set 3.5Exercise set 3.5
Exercise set 3.5
 
short course on Subsurface stochastic modelling and geostatistics
short course on Subsurface stochastic modelling and geostatisticsshort course on Subsurface stochastic modelling and geostatistics
short course on Subsurface stochastic modelling and geostatistics
 
Integration worksheet.
Integration worksheet.Integration worksheet.
Integration worksheet.
 
Midterm Study Guide
Midterm Study GuideMidterm Study Guide
Midterm Study Guide
 

Plus de Matthew Leingang

Streamlining assessment, feedback, and archival with auto-multiple-choice
Streamlining assessment, feedback, and archival with auto-multiple-choiceStreamlining assessment, feedback, and archival with auto-multiple-choice
Streamlining assessment, feedback, and archival with auto-multiple-choiceMatthew Leingang
 
Electronic Grading of Paper Assessments
Electronic Grading of Paper AssessmentsElectronic Grading of Paper Assessments
Electronic Grading of Paper AssessmentsMatthew Leingang
 
Lesson 27: Integration by Substitution (slides)
Lesson 27: Integration by Substitution (slides)Lesson 27: Integration by Substitution (slides)
Lesson 27: Integration by Substitution (slides)Matthew Leingang
 
Lesson 26: The Fundamental Theorem of Calculus (slides)
Lesson 26: The Fundamental Theorem of Calculus (slides)Lesson 26: The Fundamental Theorem of Calculus (slides)
Lesson 26: The Fundamental Theorem of Calculus (slides)Matthew Leingang
 
Lesson 26: The Fundamental Theorem of Calculus (slides)
Lesson 26: The Fundamental Theorem of Calculus (slides)Lesson 26: The Fundamental Theorem of Calculus (slides)
Lesson 26: The Fundamental Theorem of Calculus (slides)Matthew Leingang
 
Lesson 27: Integration by Substitution (handout)
Lesson 27: Integration by Substitution (handout)Lesson 27: Integration by Substitution (handout)
Lesson 27: Integration by Substitution (handout)Matthew Leingang
 
Lesson 26: The Fundamental Theorem of Calculus (handout)
Lesson 26: The Fundamental Theorem of Calculus (handout)Lesson 26: The Fundamental Theorem of Calculus (handout)
Lesson 26: The Fundamental Theorem of Calculus (handout)Matthew Leingang
 
Lesson 25: Evaluating Definite Integrals (slides)
Lesson 25: Evaluating Definite Integrals (slides)Lesson 25: Evaluating Definite Integrals (slides)
Lesson 25: Evaluating Definite Integrals (slides)Matthew Leingang
 
Lesson 25: Evaluating Definite Integrals (handout)
Lesson 25: Evaluating Definite Integrals (handout)Lesson 25: Evaluating Definite Integrals (handout)
Lesson 25: Evaluating Definite Integrals (handout)Matthew Leingang
 
Lesson 24: Areas and Distances, The Definite Integral (handout)
Lesson 24: Areas and Distances, The Definite Integral (handout)Lesson 24: Areas and Distances, The Definite Integral (handout)
Lesson 24: Areas and Distances, The Definite Integral (handout)Matthew Leingang
 
Lesson 24: Areas and Distances, The Definite Integral (slides)
Lesson 24: Areas and Distances, The Definite Integral (slides)Lesson 24: Areas and Distances, The Definite Integral (slides)
Lesson 24: Areas and Distances, The Definite Integral (slides)Matthew Leingang
 
Lesson 23: Antiderivatives (slides)
Lesson 23: Antiderivatives (slides)Lesson 23: Antiderivatives (slides)
Lesson 23: Antiderivatives (slides)Matthew Leingang
 
Lesson 23: Antiderivatives (slides)
Lesson 23: Antiderivatives (slides)Lesson 23: Antiderivatives (slides)
Lesson 23: Antiderivatives (slides)Matthew Leingang
 
Lesson 22: Optimization Problems (slides)
Lesson 22: Optimization Problems (slides)Lesson 22: Optimization Problems (slides)
Lesson 22: Optimization Problems (slides)Matthew Leingang
 
Lesson 22: Optimization Problems (handout)
Lesson 22: Optimization Problems (handout)Lesson 22: Optimization Problems (handout)
Lesson 22: Optimization Problems (handout)Matthew Leingang
 
Lesson 21: Curve Sketching (slides)
Lesson 21: Curve Sketching (slides)Lesson 21: Curve Sketching (slides)
Lesson 21: Curve Sketching (slides)Matthew Leingang
 
Lesson 21: Curve Sketching (handout)
Lesson 21: Curve Sketching (handout)Lesson 21: Curve Sketching (handout)
Lesson 21: Curve Sketching (handout)Matthew Leingang
 
Lesson 20: Derivatives and the Shapes of Curves (slides)
Lesson 20: Derivatives and the Shapes of Curves (slides)Lesson 20: Derivatives and the Shapes of Curves (slides)
Lesson 20: Derivatives and the Shapes of Curves (slides)Matthew Leingang
 
Lesson 20: Derivatives and the Shapes of Curves (handout)
Lesson 20: Derivatives and the Shapes of Curves (handout)Lesson 20: Derivatives and the Shapes of Curves (handout)
Lesson 20: Derivatives and the Shapes of Curves (handout)Matthew Leingang
 

Plus de Matthew Leingang (20)

Making Lesson Plans
Making Lesson PlansMaking Lesson Plans
Making Lesson Plans
 
Streamlining assessment, feedback, and archival with auto-multiple-choice
Streamlining assessment, feedback, and archival with auto-multiple-choiceStreamlining assessment, feedback, and archival with auto-multiple-choice
Streamlining assessment, feedback, and archival with auto-multiple-choice
 
Electronic Grading of Paper Assessments
Electronic Grading of Paper AssessmentsElectronic Grading of Paper Assessments
Electronic Grading of Paper Assessments
 
Lesson 27: Integration by Substitution (slides)
Lesson 27: Integration by Substitution (slides)Lesson 27: Integration by Substitution (slides)
Lesson 27: Integration by Substitution (slides)
 
Lesson 26: The Fundamental Theorem of Calculus (slides)
Lesson 26: The Fundamental Theorem of Calculus (slides)Lesson 26: The Fundamental Theorem of Calculus (slides)
Lesson 26: The Fundamental Theorem of Calculus (slides)
 
Lesson 26: The Fundamental Theorem of Calculus (slides)
Lesson 26: The Fundamental Theorem of Calculus (slides)Lesson 26: The Fundamental Theorem of Calculus (slides)
Lesson 26: The Fundamental Theorem of Calculus (slides)
 
Lesson 27: Integration by Substitution (handout)
Lesson 27: Integration by Substitution (handout)Lesson 27: Integration by Substitution (handout)
Lesson 27: Integration by Substitution (handout)
 
Lesson 26: The Fundamental Theorem of Calculus (handout)
Lesson 26: The Fundamental Theorem of Calculus (handout)Lesson 26: The Fundamental Theorem of Calculus (handout)
Lesson 26: The Fundamental Theorem of Calculus (handout)
 
Lesson 25: Evaluating Definite Integrals (slides)
Lesson 25: Evaluating Definite Integrals (slides)Lesson 25: Evaluating Definite Integrals (slides)
Lesson 25: Evaluating Definite Integrals (slides)
 
Lesson 25: Evaluating Definite Integrals (handout)
Lesson 25: Evaluating Definite Integrals (handout)Lesson 25: Evaluating Definite Integrals (handout)
Lesson 25: Evaluating Definite Integrals (handout)
 
Lesson 24: Areas and Distances, The Definite Integral (handout)
Lesson 24: Areas and Distances, The Definite Integral (handout)Lesson 24: Areas and Distances, The Definite Integral (handout)
Lesson 24: Areas and Distances, The Definite Integral (handout)
 
Lesson 24: Areas and Distances, The Definite Integral (slides)
Lesson 24: Areas and Distances, The Definite Integral (slides)Lesson 24: Areas and Distances, The Definite Integral (slides)
Lesson 24: Areas and Distances, The Definite Integral (slides)
 
Lesson 23: Antiderivatives (slides)
Lesson 23: Antiderivatives (slides)Lesson 23: Antiderivatives (slides)
Lesson 23: Antiderivatives (slides)
 
Lesson 23: Antiderivatives (slides)
Lesson 23: Antiderivatives (slides)Lesson 23: Antiderivatives (slides)
Lesson 23: Antiderivatives (slides)
 
Lesson 22: Optimization Problems (slides)
Lesson 22: Optimization Problems (slides)Lesson 22: Optimization Problems (slides)
Lesson 22: Optimization Problems (slides)
 
Lesson 22: Optimization Problems (handout)
Lesson 22: Optimization Problems (handout)Lesson 22: Optimization Problems (handout)
Lesson 22: Optimization Problems (handout)
 
Lesson 21: Curve Sketching (slides)
Lesson 21: Curve Sketching (slides)Lesson 21: Curve Sketching (slides)
Lesson 21: Curve Sketching (slides)
 
Lesson 21: Curve Sketching (handout)
Lesson 21: Curve Sketching (handout)Lesson 21: Curve Sketching (handout)
Lesson 21: Curve Sketching (handout)
 
Lesson 20: Derivatives and the Shapes of Curves (slides)
Lesson 20: Derivatives and the Shapes of Curves (slides)Lesson 20: Derivatives and the Shapes of Curves (slides)
Lesson 20: Derivatives and the Shapes of Curves (slides)
 
Lesson 20: Derivatives and the Shapes of Curves (handout)
Lesson 20: Derivatives and the Shapes of Curves (handout)Lesson 20: Derivatives and the Shapes of Curves (handout)
Lesson 20: Derivatives and the Shapes of Curves (handout)
 

Dernier

Moving Beyond Passwords: FIDO Paris Seminar.pdf
Moving Beyond Passwords: FIDO Paris Seminar.pdfMoving Beyond Passwords: FIDO Paris Seminar.pdf
Moving Beyond Passwords: FIDO Paris Seminar.pdfLoriGlavin3
 
From Family Reminiscence to Scholarly Archive .
From Family Reminiscence to Scholarly Archive .From Family Reminiscence to Scholarly Archive .
From Family Reminiscence to Scholarly Archive .Alan Dix
 
Generative Artificial Intelligence: How generative AI works.pdf
Generative Artificial Intelligence: How generative AI works.pdfGenerative Artificial Intelligence: How generative AI works.pdf
Generative Artificial Intelligence: How generative AI works.pdfIngrid Airi González
 
A Journey Into the Emotions of Software Developers
A Journey Into the Emotions of Software DevelopersA Journey Into the Emotions of Software Developers
A Journey Into the Emotions of Software DevelopersNicole Novielli
 
TrustArc Webinar - How to Build Consumer Trust Through Data Privacy
TrustArc Webinar - How to Build Consumer Trust Through Data PrivacyTrustArc Webinar - How to Build Consumer Trust Through Data Privacy
TrustArc Webinar - How to Build Consumer Trust Through Data PrivacyTrustArc
 
[Webinar] SpiraTest - Setting New Standards in Quality Assurance
[Webinar] SpiraTest - Setting New Standards in Quality Assurance[Webinar] SpiraTest - Setting New Standards in Quality Assurance
[Webinar] SpiraTest - Setting New Standards in Quality AssuranceInflectra
 
Potential of AI (Generative AI) in Business: Learnings and Insights
Potential of AI (Generative AI) in Business: Learnings and InsightsPotential of AI (Generative AI) in Business: Learnings and Insights
Potential of AI (Generative AI) in Business: Learnings and InsightsRavi Sanghani
 
UiPath Community: Communication Mining from Zero to Hero
UiPath Community: Communication Mining from Zero to HeroUiPath Community: Communication Mining from Zero to Hero
UiPath Community: Communication Mining from Zero to HeroUiPathCommunity
 
Rise of the Machines: Known As Drones...
Rise of the Machines: Known As Drones...Rise of the Machines: Known As Drones...
Rise of the Machines: Known As Drones...Rick Flair
 
The State of Passkeys with FIDO Alliance.pptx
The State of Passkeys with FIDO Alliance.pptxThe State of Passkeys with FIDO Alliance.pptx
The State of Passkeys with FIDO Alliance.pptxLoriGlavin3
 
Modern Roaming for Notes and Nomad – Cheaper Faster Better Stronger
Modern Roaming for Notes and Nomad – Cheaper Faster Better StrongerModern Roaming for Notes and Nomad – Cheaper Faster Better Stronger
Modern Roaming for Notes and Nomad – Cheaper Faster Better Strongerpanagenda
 
Manual 508 Accessibility Compliance Audit
Manual 508 Accessibility Compliance AuditManual 508 Accessibility Compliance Audit
Manual 508 Accessibility Compliance AuditSkynet Technologies
 
How to Effectively Monitor SD-WAN and SASE Environments with ThousandEyes
How to Effectively Monitor SD-WAN and SASE Environments with ThousandEyesHow to Effectively Monitor SD-WAN and SASE Environments with ThousandEyes
How to Effectively Monitor SD-WAN and SASE Environments with ThousandEyesThousandEyes
 
Take control of your SAP testing with UiPath Test Suite
Take control of your SAP testing with UiPath Test SuiteTake control of your SAP testing with UiPath Test Suite
Take control of your SAP testing with UiPath Test SuiteDianaGray10
 
Scale your database traffic with Read & Write split using MySQL Router
Scale your database traffic with Read & Write split using MySQL RouterScale your database traffic with Read & Write split using MySQL Router
Scale your database traffic with Read & Write split using MySQL RouterMydbops
 
So einfach geht modernes Roaming fuer Notes und Nomad.pdf
So einfach geht modernes Roaming fuer Notes und Nomad.pdfSo einfach geht modernes Roaming fuer Notes und Nomad.pdf
So einfach geht modernes Roaming fuer Notes und Nomad.pdfpanagenda
 
The Role of FIDO in a Cyber Secure Netherlands: FIDO Paris Seminar.pptx
The Role of FIDO in a Cyber Secure Netherlands: FIDO Paris Seminar.pptxThe Role of FIDO in a Cyber Secure Netherlands: FIDO Paris Seminar.pptx
The Role of FIDO in a Cyber Secure Netherlands: FIDO Paris Seminar.pptxLoriGlavin3
 
DevEX - reference for building teams, processes, and platforms
DevEX - reference for building teams, processes, and platformsDevEX - reference for building teams, processes, and platforms
DevEX - reference for building teams, processes, and platformsSergiu Bodiu
 
(How to Program) Paul Deitel, Harvey Deitel-Java How to Program, Early Object...
(How to Program) Paul Deitel, Harvey Deitel-Java How to Program, Early Object...(How to Program) Paul Deitel, Harvey Deitel-Java How to Program, Early Object...
(How to Program) Paul Deitel, Harvey Deitel-Java How to Program, Early Object...AliaaTarek5
 
Generative AI for Technical Writer or Information Developers
Generative AI for Technical Writer or Information DevelopersGenerative AI for Technical Writer or Information Developers
Generative AI for Technical Writer or Information DevelopersRaghuram Pandurangan
 

Dernier (20)

Moving Beyond Passwords: FIDO Paris Seminar.pdf
Moving Beyond Passwords: FIDO Paris Seminar.pdfMoving Beyond Passwords: FIDO Paris Seminar.pdf
Moving Beyond Passwords: FIDO Paris Seminar.pdf
 
From Family Reminiscence to Scholarly Archive .
From Family Reminiscence to Scholarly Archive .From Family Reminiscence to Scholarly Archive .
From Family Reminiscence to Scholarly Archive .
 
Generative Artificial Intelligence: How generative AI works.pdf
Generative Artificial Intelligence: How generative AI works.pdfGenerative Artificial Intelligence: How generative AI works.pdf
Generative Artificial Intelligence: How generative AI works.pdf
 
A Journey Into the Emotions of Software Developers
A Journey Into the Emotions of Software DevelopersA Journey Into the Emotions of Software Developers
A Journey Into the Emotions of Software Developers
 
TrustArc Webinar - How to Build Consumer Trust Through Data Privacy
TrustArc Webinar - How to Build Consumer Trust Through Data PrivacyTrustArc Webinar - How to Build Consumer Trust Through Data Privacy
TrustArc Webinar - How to Build Consumer Trust Through Data Privacy
 
[Webinar] SpiraTest - Setting New Standards in Quality Assurance
[Webinar] SpiraTest - Setting New Standards in Quality Assurance[Webinar] SpiraTest - Setting New Standards in Quality Assurance
[Webinar] SpiraTest - Setting New Standards in Quality Assurance
 
Potential of AI (Generative AI) in Business: Learnings and Insights
Potential of AI (Generative AI) in Business: Learnings and InsightsPotential of AI (Generative AI) in Business: Learnings and Insights
Potential of AI (Generative AI) in Business: Learnings and Insights
 
UiPath Community: Communication Mining from Zero to Hero
UiPath Community: Communication Mining from Zero to HeroUiPath Community: Communication Mining from Zero to Hero
UiPath Community: Communication Mining from Zero to Hero
 
Rise of the Machines: Known As Drones...
Rise of the Machines: Known As Drones...Rise of the Machines: Known As Drones...
Rise of the Machines: Known As Drones...
 
The State of Passkeys with FIDO Alliance.pptx
The State of Passkeys with FIDO Alliance.pptxThe State of Passkeys with FIDO Alliance.pptx
The State of Passkeys with FIDO Alliance.pptx
 
Modern Roaming for Notes and Nomad – Cheaper Faster Better Stronger
Modern Roaming for Notes and Nomad – Cheaper Faster Better StrongerModern Roaming for Notes and Nomad – Cheaper Faster Better Stronger
Modern Roaming for Notes and Nomad – Cheaper Faster Better Stronger
 
Manual 508 Accessibility Compliance Audit
Manual 508 Accessibility Compliance AuditManual 508 Accessibility Compliance Audit
Manual 508 Accessibility Compliance Audit
 
How to Effectively Monitor SD-WAN and SASE Environments with ThousandEyes
How to Effectively Monitor SD-WAN and SASE Environments with ThousandEyesHow to Effectively Monitor SD-WAN and SASE Environments with ThousandEyes
How to Effectively Monitor SD-WAN and SASE Environments with ThousandEyes
 
Take control of your SAP testing with UiPath Test Suite
Take control of your SAP testing with UiPath Test SuiteTake control of your SAP testing with UiPath Test Suite
Take control of your SAP testing with UiPath Test Suite
 
Scale your database traffic with Read & Write split using MySQL Router
Scale your database traffic with Read & Write split using MySQL RouterScale your database traffic with Read & Write split using MySQL Router
Scale your database traffic with Read & Write split using MySQL Router
 
So einfach geht modernes Roaming fuer Notes und Nomad.pdf
So einfach geht modernes Roaming fuer Notes und Nomad.pdfSo einfach geht modernes Roaming fuer Notes und Nomad.pdf
So einfach geht modernes Roaming fuer Notes und Nomad.pdf
 
The Role of FIDO in a Cyber Secure Netherlands: FIDO Paris Seminar.pptx
The Role of FIDO in a Cyber Secure Netherlands: FIDO Paris Seminar.pptxThe Role of FIDO in a Cyber Secure Netherlands: FIDO Paris Seminar.pptx
The Role of FIDO in a Cyber Secure Netherlands: FIDO Paris Seminar.pptx
 
DevEX - reference for building teams, processes, and platforms
DevEX - reference for building teams, processes, and platformsDevEX - reference for building teams, processes, and platforms
DevEX - reference for building teams, processes, and platforms
 
(How to Program) Paul Deitel, Harvey Deitel-Java How to Program, Early Object...
(How to Program) Paul Deitel, Harvey Deitel-Java How to Program, Early Object...(How to Program) Paul Deitel, Harvey Deitel-Java How to Program, Early Object...
(How to Program) Paul Deitel, Harvey Deitel-Java How to Program, Early Object...
 
Generative AI for Technical Writer or Information Developers
Generative AI for Technical Writer or Information DevelopersGenerative AI for Technical Writer or Information Developers
Generative AI for Technical Writer or Information Developers
 

Math 21a Midterm I Review

  • 1. Midterm I Review Math 21a March 5, 2008 . . Image: Flickr user Mr. Theklan . . . . . .
  • 2. Announcements ◮ Midterm covers up to Section 11.4 in text ◮ Any topics not covered in class or on HW will not be on test (i.e., curvature) ◮ Complete list of learning objectives on the Midterm I Review sheet ◮ Odd problems, chapter reviews, old exams, reviews . . . . . .
  • 3. Outline Vectors and the Geometry of Derivatives and Integrals of Space Vector Functions Three-Dimensional Arc Length (not Curvature) Coordinate Systems Motion in Space: Velocity and Vectors Acceleration The Dot Product Parametric Surfaces The Cross Product Partial Derivatives Equations of Lines and Planes Functions of Several Variables Functions and surfaces Utility Functions and Cylindrical and Spherical indifference curves Coordinates Limits and Continuity Vector Functions Partial Derivatives Vector Functions and Space Tangent Planes and Linear Curves Approximations . . . . . .
  • 4. Three-Dimensional Coordinate Systems Section 9.1 Learning Objectives ◮ To understand and be able to apply rectangular coordinate systems in R3 and the right hand rule. ◮ To understand and be able to find the distance d between two points (x1 , y1 , z1 ) and (x2 , y2 , z2 ) in R3 and to be able to use the distance formula √ d = (x1 − x2 )2 + (y1 − y2 )2 + (z1 − z2 )2 . ◮ To understand and be able to apply the equation of a sphere of radius r and center (x0 , y0 , z0 ), (x − x0 )2 + (y − y0 )2 + (z − z0 )2 = r2 . . . . . . .
  • 5. Axes in three-dimensional space z . . z y . . y . x . x . x . y . . y x . z . . z . . . . . .
  • 6. The right-hand rule z . x . . . y . . . . . . .
  • 7. Distance in space Example Find the distance between the points P1 (3, 2, 1) and P2 (4, 4, 4). . . . . . .
  • 8. Distance in space Example Find the distance between the points P1 (3, 2, 1) and P2 (4, 4, 4). z . Solution ..2 P . d . 3 . 2 . . y 1 .. √ P .1 . 5 x . √ √ d= 5 + 32 = 1+4+9 . . . . . .
  • 9. Distance in space—General Theorem The distance between (x1 , y1 , z1 ) and (x2 , y2 , z2 ) is √ (x2 − x1 )2 + (y2 − y1 )2 + (z2 − z1 )2 In space, the locus of all points which are a fixed distance from a fixed point is a sphere. . . . . . .
  • 10. Munging an equation to see its surface Example Find the surface is represented by the equation x2 + y2 + z2 − 4x + 8y − 10z + 36 = 0 . . . . . .
  • 11. Munging an equation to see its surface Example Find the surface is represented by the equation x2 + y2 + z2 − 4x + 8y − 10z + 36 = 0 Solution We can complete the square: 0 = x2 − 4x + 4 + y2 + 8y + 16 + z2 − 10z + 25 + 36 − 4 − 16 − 25 = (x − 2)2 + (y + 4)2 + (z − 5)2 − 9 So (x − 2)2 + (y + 4) + (z − 5)2 = 9 =⇒ |(x, y, z)(2, −4, 5)| = 3 This is a sphere of radius 3, centered at (2, −4, 5). . . . . . .
  • 12. Vectors Learning objectives for Section 9.2 ◮ To understand and be able to apply the definition of a scalar and a vector. ◮ To be able to represent vectors geometrically as directed line segments or algebraically as ordered pairs or triples of numbers. ◮ To understand and be able to apply the axioms or vector addition and scalar multiplication. ◮ To be understand and be able to determine the length of a vector. ◮ To understand the definitions of unit vectors, standard basis vectors, and position vectors and to be able to apply these definitions. . . . . . .
  • 13. What is a vector? Definition ◮ A vector is something that has magnitude and direction ◮ We denote vectors by boldface (v) or little arrows (⃗ One is v). good for print, one for script ◮ Given two points A and B in flatland or spaceland, the vector which starts at A and ends at B is called the displacement −→ vector AB. ◮ Two vectors are equal if they have the same magnitude and direction (they need not overlap) B . D . . v . u .. A C . . . . . . .
  • 14. Vector or scalar? Definition A scalar is another name for a real number. Example Which of these are vectors or scalars? (i) Cost of a theater ticket (ii) The current in a river (iii) The initial flight path from Boston to New York (iv) The population of the world . . . . . .
  • 15. Vector or scalar? Definition A scalar is another name for a real number. Example Which of these are vectors or scalars? (i) Cost of a theater ticket scalar (ii) The current in a river vector (iii) The initial flight path from Boston to New York vector (iv) The population of the world scalar . . . . . .
  • 16. Vector addition Definition If u and v are vectors positioned so the initial point of v is the terminal point of u, the sum u + v is the vector whose initial point is the initial point of u and whose terminal point is the terminal point of v. . u . v . v . +v u . +v u . v . u . u . . The triangle law The parallelogram law . . . . . .
  • 17. Opposite and difference Definition Given vectors u and v, ◮ the opposite of v is the vector −v that has the same length as v but points in the opposite direction ◮ the difference u − v is the sum u + (−v) . v . u . − . v . −v u . . . . . .
  • 18. Scaling vectors Definition If c is a nonzero scalar and v is a vector, the scalar multiple cv is the vector whose ◮ length is |c| times the length of v ◮ direction is the same as v if c > 0 and opposite v if c < 0 If c = 0, cv = 0. 2 .v . v −2 . 1v . . . . . . .
  • 19. Components defined Definition ◮ Given a vector a, it’s often useful to move the tail to O and measure the coordinates of the head. These are called the components of a, and we write them like this: a = ⟨a1 , a2 , a3 ⟩ or just two components if the vector is the plane. Note the angle brackets! ◮ Given a point P in the plane or space, the position vector of − → P is the vector OP. Fact − → Given points A(x1 , y1 , z1 ) and B(x2 , y2 , z2 ) in space, the vector AB has components − → AB = ⟨x2 − x1 , y2 − y1 , z2 − z1 ⟩ . . . . . .
  • 20. Vector algebra in components Theorem If a = ⟨a1 , a2 , a3 ⟩ and b = ⟨b1 , b2 , b3 ⟩, and c is a scalar, then ◮ a + b = ⟨a1 + b1 , a2 + b2 , a3 + b3 ⟩ ◮ a − b = ⟨a1 − b1 , a2 − b2 , a3 − b3 ⟩ ◮ ca = ⟨ca1 , ca2 , ca3 ⟩ . . . . . .
  • 21. Properties Theorem Given vectors a, b, and c and scalars c and d, we have 1. a+b=b+a 6. (c + d)a = ca + da 2. a + (b + c ) = ( a + b ) + c 7. (cd)a = c(da) 3. a+0=a 4. a + (−a) = 0 8. 1a = a 5. c(a + b) = ca + cb These can be verified geometrically. . . . . . .
  • 22. Length Definition Given a vector v, its length is the distance between its initial and terminal points. Fact The length of a vector is the square root of the sum of the squares of its components: √ |⟨a1 , a2 , a3 ⟩| = a2 + a2 + a2 1 2 3 . . . . . .
  • 23. Useful vectors Definition ◮ A unit vector is a vector whose length is one ◮ We define the standard basis vectors i = ⟨1, 0, 0⟩, j = ⟨0, 1, 0⟩, k = ⟨0, 0, 1⟩. In script, they’re often written as ˆ, ȷ, ı ˆ ˆ k. Fact Any vector a can be written as a linear combination of the standard basis vectors ⟨a1 , a2 , a3 ⟩ = a1 i + a2 j + a3 k. . . . . . .
  • 24. The Dot Product I Learning objectives for Section 9.3 ◮ To understand and be able to use the definitions of the dot product to measure the length of a vector and the angle θ between two vectors. Given two vectors a = ⟨a1 , a2 , a3 ⟩ and b = ⟨b1 , b2 , b3 ⟩, a · b = |a||b| cos θ = a1 b1 + a2 b2 + a3 b3 . ◮ To understand and be able to apply properties of the dot product. . . . . . .
  • 25. The Dot Product II Learning objectives for Section 9.3 ◮ To understand that two vectors are orthogonal if their dot product is zero. ◮ To understand and be able to determine the projection of a vector a onto a vector b, a·b proja b = a |a|2 ◮ To be able to use vectors and the dot product in applications. . . . . . .
  • 26. Definition If a and b are any two vectors in the plane or in space, the dot product (or scalar product) between them is the quantity a · b = |a| |b| cos θ, where θ is the angle between them. Another way to say this is that a · b is |b| times the length of the projection of a onto b. . a . · b is |b| times this length. a . b In components, if a = ⟨a1 , a2 , a3 ⟩ and b = ⟨b1 , b2 , b3 ⟩, then a · b = a1 b1 + a2 b2 + a3 b3. . . . . .
  • 27. Algebraic properties of the dot product Fact If a, b and c are vectors are c is a scalar, then 1. a · a = |a|2 4. (ca) · b = c(a · b) = a · (cb) 2. a · b = b · a 5. 0 · a = 0 3. a · (b + c) = a · b + a · c . . . . . .
  • 28. Geometric properties of the dot product Fact The projection of b onto a is given by a·b proja b = a |a|2 Fact The angle between two nonzero vectors a and b is given by a·b cos θ = , |a| |b| where θ is taken to be between 0 and π . . . . . . .
  • 29. More geometric properties of the dot product Fact The angle between two nonzero vectors a and b is acute if a · b > 0. obtuse if a · b < 0, right if a · b = 0. The vectors are parallel if a · b = ± |a| |b|. ◮ b is a positive multiple of a if a · b = |a| |b| ◮ b is a negative multiple of a if a · b = − |a| |b| . . . . . .
  • 30. Other uses of the dot product ◮ similarity ◮ Work W=F·d . . . . . .
  • 31. The Cross Product Learning objectives for Section 9.4 ◮ To understand and be able to use the definitions of the cross product to find a vector that is orthogoal two vectors. Given two vectors a = ⟨a1 , a2 , a3 ⟩ and b = ⟨b1 , b2 , b3 ⟩, a × b = (|a||b| sin θ)n = ⟨a2 b3 − a3 b2 , a3 b1 − a1 b3 , a1 b2 − a2 b1 ⟩, where n is a unit vector perpendicular to both a and b. ◮ To understand and be able to apply properties of the dot product, especially a × b = −b × a. ◮ To understand that two vectors are parallel if and only if their cross product is zero. ◮ To be able to use vectors and the cross product in applications. . . . . . .
  • 32. Definition of the cross product Definition Given vectors a and b in space, the cross product of a and b is the vector a × b = |a| |b| (sin θ) n, where n is a vector perpendicular to a and b such that (a, b, n) is a right-handed set of three vectors. In components, if a = ⟨a1 , a2 , a3 ⟩= a1 i + a2 j + a3 k b = ⟨b1 , b2 , b3 ⟩= b1 i + b2 j + b3 k Then a × b = (a2 b3 − b2 a3 )i + (a3 b1 − b3 a1 )j + (a1 b2 − b1 a2 )k = ⟨a 2 b 3 − b 2 a 3 , a 3 b 1 − b 3 a 1 , a 1 b 2 − b 1 a 2 ⟩ . . . . . .
  • 33. Determinant formula This is only to help you remember, in case you’ve seen determinants of 3 × 3 matrices: i j k a a a a a a a1 a2 a3 = i 2 3 − j 1 3 + k 1 2 b2 b3 b1 b 3 b1 b2 b1 b2 b3 = (a2 b3 − b2 a3 )i − (b3 a1 − a3 b1 )j + (a1 b2 − b1 a2 )k =a×b . . . . . .
  • 34. Algebraic Properties of the Cross Product If a, b, and c are vectors and c is a scalar, then 1. a × b = −b × a 2. (ca) × b = c(a × b) = a × (cb) 3. a × (b + c) = a × b + a × c 4. (a + b) × c = a × c + b × c . . . . . .
  • 35. Geometric Properties of the cross product ◮ a × b = 0 exactly when a and b are parallel. ◮ The magnitude of the cross product a × b is the area of the parallelogram with sides a and b. . b | . b| sin θ θ . . . a . . . . . .
  • 36. Applications of the cross product ◮ Area of a parallelogram ◮ Torque τ = r × F ◮ Volume of a parallelipiped (scalar triple product) V = |a · (b × c)| . . . . . .
  • 37. The scalar triple product in action Example Find the volume of the parallelipiped spanned by a = ⟨2, 0, −3⟩, b = ⟨1, 1, 1⟩, and c = ⟨0, 4, −1⟩ . . . . . .
  • 38. The scalar triple product in action Example Find the volume of the parallelipiped spanned by a = ⟨2, 0, −3⟩, b = ⟨1, 1, 1⟩, and c = ⟨0, 4, −1⟩ Solution The scalar triple product can be calculated by . b 2 0 −3 . c 1 1 1 = −22 . 0 4 −1 It’s negative because the triple . a (a, b, c) is left-handed. So the volume is 22. . . . . . .
  • 39. Equations of Lines and Planes I ◮ To understand and be able to represent a line ℓ in R3 using ◮ the vector equation r = r0 + tv ◮ the parametric equations x = x0 + at y = y0 + bt z = z0 + ct ◮ the symmetric equations x − x0 y − y0 z − z0 = = a b c . . . . . .
  • 40. Equations of Lines and Planes II ◮ To be able to represent line segment in R3 from r0 to r1 , r(t) = (1 − t)r0 + tr1 , where 0 ≤ t ≤ 1. ◮ To understand and be able to represent a plane in R3 given a normal vector n = ⟨a, b, c⟩ and a point P0 = (x0 , y0 , z0 ) in the plane, n · ⟨x − x 0 , y − y 0 , z − z 0 ⟩ = 0 or a(x − x0 ) + b(y − y0 ) + c(z − z0 ) = 0. . . . . . .
  • 41. Equations of Lines and Planes III ◮ To understand and be able to calculate the distance D from a point P1 = (x1 , y1 , z1 ) to the plane ax + by + cz + d = 0, |ax1 + by1 + cz1 + d| D= √ . a 2 + b2 + c2 . . . . . .
  • 42. Applying the definition Example Find the vector, parametric, and symmetric equations for the line that passes through (1, 2, 3) and (2, 3, 4). . . . . . .
  • 43. Applying the definition Example Find the vector, parametric, and symmetric equations for the line that passes through (1, 2, 3) and (2, 3, 4). Solution ◮ Use the initial vector ⟨1, 2, 3⟩ and direction vector ⟨2, 3, 4⟩ − ⟨1, 2, 3⟩ = ⟨1, 1, 1⟩. Hence r(t) = ⟨1, 2, 3⟩ + t ⟨1, 1, 1⟩ ◮ The parametric equations are x=1+t y=2+t z=3+t ◮ The symmetric equations are x−1=y−2=z−3 . . . . . .
  • 44. Planes The set of all points whose displacement vector from a fixed point is perpendicular to a fixed vector is a plane. z . . n .0 r . y . . x . . . . . .
  • 45. Equations for planes The plane passing through the point with position vector r0 = ⟨x0 , y0 , z0 ⟩ perpendicular to ⟨a, b, c⟩ has equations: ◮ The vector equation n · (r − r0 ) = 0 ⇐⇒ n · r = n · r0 ◮ Rewriting the dot product in component terms gives the scalar equation a(x − x0 ) + b(y − y0 ) + c(z − z0 ) = 0 The vector n is called a normal vector to the plane. ◮ Rearranging this gives the linear equation ax + by + cz + d = 0, where d = −ax0 − by0 − cz0 . . . . . . .
  • 46. Example Find an equation of the plane that passes through the points P(1, 2, 3), Q(3, 5, 7), and R(4, 3, 1). . . . . . .
  • 47. Example Find an equation of the plane that passes through the points P(1, 2, 3), Q(3, 5, 7), and R(4, 3, 1). Solution− → − → − → Let r0 = OP = ⟨1, 2, 3⟩. To get n, take PQ × PR: i j k − → − → PQ × PR = 2 3 4 = ⟨−10, 16, −7⟩ 3 1 −2 So the scalar equation is −10(x − 1) + 16(y − 2) − 7(z − 3) = 0. . . . . . .
  • 48. Distance from point to line Definition The distance between a point and a line is the smallest distance from that point to all points on the line. You can find it by projection. Q . . b·v .b − v v·v . b b·v . rojv b = p v . v v·v θ . . P .0 . . . . . .
  • 49. Distance from a point to a plane Definition The distance between a point and a plane is the smallest distance from that point to all points on the line. . . Q . b |n · b| . |n| . n . P .0 To find the distance from the a point to a plane, project the displacement vector from any point on the plane to the given point onto the normal vector. . . . . . .
  • 50. Distance from a point to a plane II We have |n · b| D= |n| If Q = (x1 , y1 , z1 ), and the plane is given by ax + by + cz + d = 0, then n = ⟨a, b, c⟩, and n · b = ⟨a, b, c⟩ · ⟨x1 − x0 , y1 − y0 , z1 − z0 ⟩ = ax1 + by1 + cz1 − ax0 − by0 − cz0 = ax1 + by1 + cz1 + d So the distance between the plane ax + by + cz + d = 0 and the point (x1 , y1 , z1 ) is |ax1 + by1 + cz1 + d| D= √ a 2 + b 2 + c2 . . . . . .
  • 51. Functions and surfaces Learning objectives for Section 9.6 ◮ To understand and be able to represent a function of two variables, z = f(x, y), graphically. Section 9.6) ◮ To understand and be able to use the trace (cross-section) of a function. Section 9.6) ◮ To understand and be able to determine the domain of a function z = f(x, y). Section 9.6) ◮ To understand and be able to represent a selection of quadric surfaces graphically. . . . . . .
  • 52. function, domain, range Definition A function f of two variables is a rule that assigns to each ordered pair of real numbers (x, y) in a set D a unique real number denoted by f(x, y). The set D is the domain of f and its range is the set of values that f takes on. That is range f = { f(x, y) | (x, y) ∈ D } . . . . . .
  • 53. Example Example √ Find the domain and range of f(x, y) = xy. . . . . . .
  • 54. Example Example √ Find the domain and range of f(x, y) = xy. Solution ◮ Working from the outside in, we see that xy must be nonnegative, which means x ≥ 0 and y ≥ 0 or x ≤ 0 and y ≤ 0. Thus the domain is the union of the coordinate axes, and the first and third quadrants. ◮ The range of f is the set of all “outputs” of f. Clearly the range of f is restricted to the set of nonnegative numbers. To make sure that we can get all nonnegative numbers x, notice x = f(x, x). . . . . . .
  • 55. Solution continued Here is a sketch of the domain: . . . . . . .
  • 56. Traces and surfaces Example Describe and sketch the surfaces. (i) z = 4x2 + y2 (vi) y2 − 9x2 − 4z2 = 36 (ii) z = 4 − x 2 − y2 (vii) x2 + 4y2 + 2z2 = 4 (iii) z 2 = 4 (x 2 + y 2 ) (iv) x = 2y2 − z2 (viii) y2 + z2 = 1 (v) x2 + y2 − 9z2 = 9 . . . . . .
  • 57. Sketch of z = 4x2 + y2 The trace in the xz-plane (y = 0) is the parabola z = 4x2 . The trace in the yz-plane is the parabola z = y2 . The trace in the xy-plane is the curve 4x2 + y2 = 0, which is just a point (0, 0). But the trace in the plane z = k (k > 0) is the ellipse 4x2 + y2 = k. So we get ellipses crossing the parabolas, an elliptic paraboloid. 2 1 0 1 2 2.0 1.5 1.0 0.5 0.0 1.0 0.5 0.0 0.5 1.0 . . . . . .
  • 58. Sketch of z2 = 4(x2 + y2 ) The trace in the xz-plane is the equation z2 = 4x2 , or z = ±2x. The trace in the yz-plane is the equation z = ±2y. Traces in the plane z = k are circles k2 = 4(x2 + y2 ). So we have a cone. 1.0 0.5 0.0 0.5 1.0 1.0 0.5 0.0 0.5 1.0 1.0 0.5 0.0 0.5 1.0 . . . . . .
  • 59. Sketch of x = 2y2 − z2 The trace in the xy-plane is a parabola x = 2y2 . The trace in the xz-plane is a parabola opening the other direction: x = −z2 . Traces x = k give hyperbolas. So we have a hyperbolic paraboloid. 1.0 0.5 0.0 0.5 1.0 1.0 0.5 0.0 0.5 1.0 1.0 0.5 0.0 0.5 1.0 . . . . . .
  • 60. Sketch of x2 + y2 − 9z2 = 9 The x2 + y2 piece tells us that this is a surface of revolution, where the z-axis is the axis of revolution. If we find the xz-trace, we get x2 − 9z2 = 9, a hyperbola. We get a hyperboloid of one sheet. 2 1 0 4 1 2 2 4 0 2 0 2 2 4 4 . . . . . .
  • 61. Sketch of y2 − 9x2 − 4z2 = 36 Traces in the xy and yz planes are hyperbolas. Traces in the plane y = k are ellipses 9x2 + 4z2 = k2 − 36. We have a hyperboloid of two sheets. 10 5 0 5 10 4 2 0 2 4 2 0 2 . . . . . .
  • 62. Sketch of x2 + 4y2 + 2z2 = 4 The traces in the planes x = 0, y = 0, and z = 0 are all ellipses. We get an ellipsoid. 1.0 2 0.5 1 0.0 0 0.5 1 1.0 2 2 1 0 1 2 . . . . . .
  • 63. Traces and surfaces Example Describe and sketch the surfaces. (i) z = 4x2 + y2 elliptic (v) x2 + y2 − 9z2 = 9 paraboloid hyperboloid of one sheet (ii) z = 4 − x2 − y2 elliptic (vi) y2 − 9x2 − 4z2 = 36 paraboloid hyperboloid of two sheets (iii) z2 = 4(x2 + y2 ) cone (vii) x2 + 4y2 + 2z2 = 4 ellipsoid (iv) x = 2y2 − z2 hyperbolic (viii) y2 + z2 = 1 cylinder paraboloid . . . . . .
  • 64. Cylindrical and Spherical Coordinates I Learning objectives for Section 9.7 ◮ To understand and be able to apply the polar coordinate system in R2 , and to understand the relationship to rectangular coordinates: x = r cos θ y = r sin θ 2 2 2 r =x +y tan θ = y/x, where r ≥ 0 and 0 ≤ θ ≤ 2π . . . . . . .
  • 65. Cylindrical and Spherical Coordinates II Learning objectives for Section 9.7 ◮ To understand and be able to apply the cylindrical coordinate system in R3 , and to understand the relationship to rectangular coordinates: x = r cos θ y = r sin θ z=z r 2 = x2 + y 2 tan θ = y/x z = z, where r ≥ 0 and 0 ≤ θ ≤ 2π . . . . . . .
  • 66. Cylindrical and Spherical Coordinates III Learning objectives for Section 9.7 ◮ To understand and be able to apply the spherical coordinate system in R3 , and to understand the relationship to rectangular coordinates: x = ρ sin φ cos θ y = ρ sin φ sin θ z = ρ cos φ 2 2 2 2 ρ =x +y +z , where r ≥ 0, 0 ≤ θ ≤ 2π , and 0 ≤ φ ≤ π . . . . . . .
  • 67. Why different coordinate systems? ◮ The dimension of space comes from nature ◮ The measurement of space comes from us ◮ Different coordinate systems are different ways to measure space . . . . . .
  • 68. Vectors and the Geometry of Space Conversion from polar to cartesian (rectangular) x = r cos θ y = r sin θ Conversion from cartesian to . r . y θ polar: . . . x √ r = x2 + y 2 x y y cos θ = sin θ = tan θ = r r x . . . . . .
  • 69. Cylindrical Coordinates Just add the vertical dimension Conversion from cylindrical to cartesian (rectangular): z . x = r cos θ y = r sin θ ( . r, θ, z) z=z Conversion from cartesian to z z . . cylindrical: . r . . y √ θ . y . . r = x2 + y 2 x . . x y y x . cos θ = sin θ = tan θ = r r x z=z . . . . . .
  • 70. Spherical Coordinates like the earth, but not exactly Conversion from spherical to cartesian (rectangular): z . x = ρ sin φ cos θ . y = ρ sin φ sin θ ( . r, θ, φ) z = ρ cos φ . Conversion from cartesian to φ ρ . . z . spherical: . . y √ √ x . θ . r = x2 + y2 ρ = x2 + y2 + z2 y . x y y x . cos θ = sin θ = tan θ = r r x z cos φ = ρ . . . . . .
  • 71. Outline Vectors and the Geometry of Derivatives and Integrals of Space Vector Functions Three-Dimensional Arc Length (not Curvature) Coordinate Systems Motion in Space: Velocity and Vectors Acceleration The Dot Product Parametric Surfaces The Cross Product Partial Derivatives Equations of Lines and Planes Functions of Several Variables Functions and surfaces Utility Functions and Cylindrical and Spherical indifference curves Coordinates Limits and Continuity Vector Functions Partial Derivatives Vector Functions and Space Tangent Planes and Linear Curves Approximations . . . . . .
  • 72. Vector Functions and Space Curves Learning objectives for Section 10.1 ◮ To understand and be able to apply the concept of a vector-valued function, r(t) = ⟨f(t), g(t), h(t)⟩ = f(t)i + g(t)j + h(t)k. ◮ To be able to apply the concepts of limits and continuity to vector-valued functions. ◮ To understand that a curve in R3 can be represented parametrically x = f(t) y = g(t) z = h (t) or by a vector-valued function r(t) = ⟨f(t), g(t), h(t)⟩ . . . . . .
  • 73. Example Given the plane curve described by the vector equation r(t) = sin(t)i + 2 cos(t)j (a) Sketch the plane curve. . . . . . .
  • 74. Solution r(t) = r(t) = sin(t)i + 2 cos(t)j y . t r (t) 0 2j π/2 i . (π/4) r π −2j . x . 3π/2 −i 2π 2j . . . . . .
  • 75. Derivatives and Integrals of Vector Functions Learning objectives for Section 10.2 ◮ To understand and be able to calculate the derivative of a vector-valued function. ◮ To understand and be able to apply the basic rules of differentiation for vector-valued functions. ◮ To understand and be able to find the definite integral of a vector-valued function. . . . . . .
  • 76. Derivatives of vector-valued functions Definition Let r be a vector function. ◮ The limit of r at a point a is defined componentwise: ⟨ ⟩ lim r(t) = lim f(t), lim g(t), lim h(t) t→a t→a t→a t→a ◮ The derivative of r is defined in much the same way as it is for real-valued functions: dr r(t + h) − r(t) = r′ (t) = lim dt h→0 h . . . . . .
  • 77. Rules for differentiation Theorem Let u and v be differentiable vector functions, c a scalar, and f a real-valued function. Then: d 1. [u(t) + v(t)] = u′ (t) + v′ (t) dt d 2. [cu(t)] = cu′ (t) dt d 3. [f(t)u(t)] = f′ (t)u(t) + f(t)u′ (t) dt d 4. [u(t) · v(t)] = u′ (t) · v(t) + u(t) · v′ (t) dt d 5. [u(t) × v(t)] = u′ (t) × v(t) + u(t) × v′ (t) dt d 6. [u(f(t))] = f′ (t)u′ (f(t)) dt . . . . . .
  • 78. Example Given the plane curve described by the vector equation r(t) = sin(t)i + 2 cos(t)j (a) Sketch the plane curve. (b) Find r′ (t) . . . . . .
  • 79. Solution r(t) = r(t) = sin(t)i + 2 cos(t)j r′ (t) = cos(t)i − 2 sin(t)j y . t r (t) 0 2j π/2 i . (π/4) r π −2j . x . 3π/2 −i 2π 2j . . . . . .
  • 80. Example Given the plane curve described by the vector equation r(t) = sin(t)i + 2 cos(t)j (a) Sketch the plane curve. (b) Find r′ (t) (c) Sketch the position vector r(π/4) and the tangent vector r′ (π/4). . . . . . .
  • 81. Solution r(t) = r(t) = sin(t)i + 2 cos(t)j r′ (t) = cos(t)i − 2 sin(t)j y . t r (t) 0 2j π/2 i . (π/4) r . ′ (π/4) r π −2j . . x 3π/2 −i 2π 2j . . . . . .
  • 82. Integrals of vector-valued functions Definition Let r be a vector function defined on [a, b]. For each whole number n, divide the interval [a, b] into n pieces of equal width ∆t. Choose a point t∗ on each subinterval and form the Riemann sum i ∑ n Sn = r(t∗ ) ∆t i i=1 Then define ∫ b ∑ n r(t) dt = lim Sn = lim r(t∗ ) ∆t i a n→∞ n→∞ i=1 [ ] ∑ n ∑ n ∑ n = lim f(t∗ ) ∆ti i + g(t∗ ) ∆tj i + h(t∗ ) ∆tk i n→∞ i=1 i=1 i=1 (∫ b ) (∫ b ) (∫ b ) = f(t) dt i + g(t) dt j + h(t) dt k a a a . . . . . .
  • 83. FTC for vector functions Theorem (Second Fundamental Theorem of Calculus) If r(t) = R′ (t), then ∫ b r(t) dt = R(b) − R(a) a Example ∫ π Given r(t) = ⟨t, cos 2t, sin 2t⟩, find r(t) dt. 0 Answer ⟨∫ π ∫ π ∫ π ⟩ ⟨ ⟩ π2 t dt, cos 2t dt, sin 2t dt = , 0, 0 0 0 0 2 . . . . . .
  • 84. Arc Length (not Curvature) Learning objectives for Section 10.3 ◮ To understand and be able to apply the arc length of a curve, r(t) = ⟨f(t), g(t), h(t)⟩, over an interval a ≤ t ≤ b, ∫ b ∫ b√ ′ L= |r (t)| dt = [f′ (t)]2 + [g′ (t)]2 + [h′ (t)]2 dt. a a ◮ To understand and be able to apply the arc length function of a curve, r(t) = ⟨f(t), g(t), h(t)⟩, over an interval a ≤ t ≤ b, ∫ t ∫ t√ ′ s (t) = |r (u)| du = [f′ (u)]2 + [g′ (u)]2 + [h′ (u)]2 du. a a . . . . . .
  • 85. Length of a curve Break up the curve into pieces, and approximate the arc length with the sum of the lengths of the pieces: y . . . x . . . . . .
  • 86. Length of a curve Break up the curve into pieces, and approximate the arc length with the sum of the lengths of the pieces: y . . . x . . . . . .
  • 87. Length of a curve Break up the curve into pieces, and approximate the arc length with the sum of the lengths of the pieces: y . . . x . . . . . .
  • 88. Length of a curve Break up the curve into pieces, and approximate the arc length with the sum of the lengths of the pieces: y . . . x . . . . . .
  • 89. Length of a curve Break up the curve into pieces, and approximate the arc length with the sum of the lengths of the pieces: y . . . x . . . . . .
  • 90. Length of a curve Break up the curve into pieces, and approximate the arc length with the sum of the lengths of the pieces: y . ∑√ n . ≈ L (∆xi )2 + (∆yi )2 i=1 . x . . . . . . .
  • 91. Sum goes to integral If ⟨x, y⟩ is given by a vector-valued function r(t) = ⟨f(t), g(t), ⟩ with domain [a, b], we can approximate: ∆xi ≈ f′ (ti )∆ti ∆xi ≈ g′ (ti )∆ti So ∑√ n ∑ √[ n ]2 L≈ (∆xi )2 + (∆yi )2 ≈ f′ (ti )∆ti + [g′ (ti )∆ti ]2 i=1 i=1 ∑ √[ n ]2 = f′ (ti ) + [g′ (ti )]2 ∆ti i=1 As n → ∞, this converges to ∫ b√ L= [f′ (t)]2 + [g′ (t)]2 dt a In 3D, r(t) = ⟨f(t), g(t), h(t)⟩, and ∫ b√ L= [f′ (t)]2 + [g′ (t)]2 + [h′ (t)]2 dt a . . . . . .
  • 92. Example Example Find the length of the parabola y = x2 from x = 0 to x = 1. . . . . . .
  • 93. Example Example Find the length of the parabola y = x2 from x = 0 to x = 1. Solution ⟨ ⟩ Let r(t) = t, t2 . Then ∫ 1√ √ 5 1 √ L= 1 + (2t)2 = + ln 2 + 5 0 2 4 . . . . . .
  • 94. Motion in Space: Velocity and Acceleration Learning objectives for Section 10.4 ◮ To understand the first derivative as the velocity of a curve and the second derivative as the acceleration of a curve and be able to apply these definitions. . . . . . .
  • 95. Velocity and Acceleration Definition Let r(t) be a vector-valued function. ◮ The velocity v(t) is the derivative r′ (t) ◮ The speed is the length of the derivative |r′ (t)| ◮ The acceleration is the second derivative r′′ (t). . . . . . .
  • 96. Example Find the velocity, acceleration, and speed of a particle with position function r(t) = ⟨2 sin t, 5t, 2 cos t⟩ . . . . . .
  • 97. Example Find the velocity, acceleration, and speed of a particle with position function r(t) = ⟨2 sin t, 5t, 2 cos t⟩ Answer ◮ r′ (t) = ⟨2 cos(t), 5, −2 sin(t)⟩ √ ◮ r′ (t) = 29 ◮ r′′ (t) = ⟨−2 sin(t), 0, −2 cos(t)⟩ . . . . . .
  • 98. Parametric Surfaces Learning objectives for Section 10.5 ◮ To understand and be able to apply the concept of a parametric surface. A parametric surface may be defined by a vector equation, r(t) = x(u, v) i + y(u, v) j + z(u, v) k or by a set of parametric equations x = x (u , v ) y = y(u, v) z = z(u, v), where u and v are variables with a domain D contained in R2 . ◮ To understand and be able to represent surfaces of revolutions parametrically. . . . . . .
  • 99. Surfaces with easy parametrizations ◮ graph ◮ plane ◮ sphere ◮ surface of revolution . . . . . .
  • 100. Parametrizing a graph Example Parametrize the surface described by z = x 2 + y2 , −2 ≤ x ≤ 2 , −3 ≤ y ≤ 3 . . . . . .
  • 101. Parametrizing a graph Example Parametrize the surface described by z = x 2 + y2 , −2 ≤ x ≤ 2 , −3 ≤ y ≤ 3 Solution Let x = u, y = v, z = u 2 + v2 on the domain −2 ≤ u ≤ 2, −3 ≤ v ≤ 3. . . . . . .
  • 102. parametrizing a sphere Example Find a parametrization for the unit sphere. . . . . . .
  • 103. parametrizing a sphere Example Find a parametrization for the unit sphere. Solution Use spherical coordinates. Let: x = cos θ sin φ, y = sin θ sin φ, z = cos φ The domain of parametrization is 0 ≤ θ ≤ 2π , 0 ≤ φ ≤ π . . . . . . .
  • 104. Parametrizing a surface of revolution Example Find a parametrization of the surface described by x2 − y 2 + z 2 = 1 , −3 ≤ y ≤ 3 . . . . . .
  • 105. Parametrizing a surface of revolution Example Find a parametrization of the surface described by x2 − y 2 + z 2 = 1 , −3 ≤ y ≤ 3 Solution The surface is the graph of z2 − y2 = 1 revolved around the y-axis. So let √ √ x= 1+y 2 cos θ, y = u, z = 1 + y2 sin θ The domain is −3 ≤ u ≤ 3, 0 ≤ θ ≤ 2π . . . . . . .
  • 106. Outline Vectors and the Geometry of Derivatives and Integrals of Space Vector Functions Three-Dimensional Arc Length (not Curvature) Coordinate Systems Motion in Space: Velocity and Vectors Acceleration The Dot Product Parametric Surfaces The Cross Product Partial Derivatives Equations of Lines and Planes Functions of Several Variables Functions and surfaces Utility Functions and Cylindrical and Spherical indifference curves Coordinates Limits and Continuity Vector Functions Partial Derivatives Vector Functions and Space Tangent Planes and Linear Curves Approximations . . . . . .
  • 107. Functions of Several Variables Learning objectives for Section 11.1 ◮ To understand functions of several variables and be able to represent these functions using level sets. . . . . . .
  • 108. A contour plot is a topographic map of a graph √ Consider the graph z = x2 + y2 Intersect the cone with planes z = c and what do you get? Circles. A contour plot shows evenly spaced circles. 3 2 1 0 4 3 -1 2 2 1 0 -2 0 -2 0 -2 -3 -3 -2 -1 0 1 2 3 2 . . . . . .
  • 109. The paraboloid Example Graph z = x2 + y2 . 3 2 1 0 15 10 -1 2 5 0 0 -2 -2 0 -2 -3 -3 -2 -1 0 1 2 3 2 . . . . . .
  • 110. The hyperbolic paraboloid Example Graph z = x2 − y2 . 3 2 1 0 5 -1 0 2 -5 -2 0 -2 0 -2 -3 -3 -2 -1 0 1 2 3 2 . . . . . .
  • 111. Plotting a Cobb-Douglas function Example Plot z = x1/2 y1/2 . 3 2.5 2 1.5 3 2 3 1 1 2 0 0 0.5 1 1 2 0 0 0.5 1 1.5 2 2.5 3 30 . . . . . .
  • 112. Utility Functions and indifference curves ◮ If u is a utility function, a level curve of u is a curve along which all points have the same u value. ◮ We also know this as an indifference curve . . . . . .
  • 113. Limits and Continuity Learning objectives for Section 11.2 ◮ To understand and be able to apply the concept of a limit of a function of several variables. ◮ To understand and be able to apply the definition of continuity for a function of several variables. . . . . . .
  • 114. Definition We write lim f(x, y) = L (x,y)→(a,b) and we say that the limit of f(x, y) as (x, y) approaches (a, b) is L if we can make the values of f(x, y) as close to L as we like by taking the point (x, y) to be sufficiently close to (a, b). . . . . . .
  • 115. easy limits ◮ lim x=a (x,y)→(a,b) ◮ lim y=b (x,y)→(a,b) ◮ lim c=c (x,y)→(a,b) . . . . . .
  • 116. Like regular limits, limits of multivariable functions can be ◮ added ◮ subtracted ◮ multiplied ◮ composed ◮ divided, provided the limit of the denominator is not zero. . . . . . .
  • 117. Limit of a Polynomial Example Find lim (x5 + 4x3 y − 5xy2 ) (x,y)→(5,−2) . . . . . .
  • 118. Limit of a Polynomial Example Find lim (x5 + 4x3 y − 5xy2 ) (x,y)→(5,−2) Solution lim (x5 + 4x3 y − 5xy2 ) = (5)5 + 4(5)3 (−2) − 5(5)(−2)2 (x,y)→(5,−2) = 3125 + 4(125)(−2) − 5(5)(4) = 2025. . . . . . .
  • 119. Limit of a Rational Expression Example Compute x2 lim . (x,y)→(1,2) x2 + y2 . . . . . .
  • 120. Limit of a Rational Expression Example Compute x2 lim . (x,y)→(1,2) x2 + y2 Solution x2 (1)2 lim = (x,y)→(1,2) x2 + y2 (1)2 + (2)2 1 = 5 . . . . . .
  • 121. What can go wrong? The only real problem is a limit where the denominator goes to zero. ◮ If the numerator goes to some number and the denominator goes to zero then the quotient cannot have a limit. . . . . . .
  • 122. What can go wrong? The only real problem is a limit where the denominator goes to zero. ◮ If the numerator goes to some number and the denominator goes to zero then the quotient cannot have a limit. ◮ If on the other hand the numerator and denominator both go to zero we have no clue. . . . . . .
  • 123. Showing a limit doesn’t exist Theorem Suppose lim f(x, y) = L. Then the limit of f as (x, y) → (a, b) is L (x,y)→(a,b) along all paths through (a, b). There are two contrapositives to this statement: ◮ If there is a path through (a, b) along which the limit does not exist, the two-dimensional limit does not exist ◮ If there are two paths through (a, b) along which the limits exist but disagree, the two-dimensional limit does not exist . . . . . .
  • 124. Continuity Definition A function f of two variables is called continuous at (a, b) if lim f(x, y) = f(a, b). (x,y)→(a,b) We say f is continuous on D if f is continuous at every point (a, b) in D. . . . . . .
  • 125. Partial Derivatives Learning objectives for Section 11.3 ◮ To understand and be able to apply the definition of a partial derivative. ◮ To be able to compute partial derivatives. ◮ To understand and be able to apply Clairaut’s Theorem. If f is defined on a disk D that contains the point (a, b) and the functions fxy and fyx are continuous on D, then fxy (a, b) = fyx (a, b). ◮ To understand the idea of a partial differential equation, and to be able to verify solutions to partial differential equations. . . . . . .
  • 126. Definition Let f(x, y) be a function of two variables. We define the partial ∂f ∂f derivatives and at a point (a, b) as ∂x ∂y ∂f f (a + h, b) − f (a, b) (a, b) = lim ∂x h→0 h ∂f f (a, b + h) − f (a, b) (a, b) = lim ∂y h→0 h In other words, we temporarily treat the other variable as constant and differentiate the resulting one-variable function as in Calculus I. . . . . . .
  • 127. Example Let f(x, y) = x3 − 3xy2 . Find its partial derivatives. . . . . . .
  • 128. Example Let f(x, y) = x3 − 3xy2 . Find its partial derivatives. Solution ∂f When finding , we hold y constant. So ∂x ∂f ∂ = 3x2 − (3y2 ) (x) = 3x2 − 3y2 ∂x ∂x Similarly, ∂f = 0 − 3x(2y) = −6xy ∂y . . . . . .
  • 129. Second derivatives If f(x, y) is a function of two variables, each of its partial derivatives are function of two variables, and we can hope that they are differentiable, too. So we define the second partial derivatives. ( ) ∂2f ∂ ∂f = = fxx ∂ x2 ∂x ∂x ( ) ∂2f ∂ ∂f = = fxy ∂y ∂x ∂y ∂x ( ) ∂2f ∂ ∂f = = fyx ∂x ∂y ∂x ∂y ( ) ∂2f ∂ ∂f = = fyy ∂ y2 ∂y ∂y . . . . . .
  • 130. Don’t worry about the mixed partials The “mixed partials” bookkeeping may seem scary. However, we are saved by: Theorem (Clairaut’s Theorem/Young’s Theorem) If f is defined near (a, b) and fxy and fyx are continuous at (a, b), then fxy (a, b) = fyx (a, b). The upshot is that we needn’t worry about the ordering. . . . . . .
  • 131. Example (Continued) Let f(x, y) = x3 − 3xy2 . Find the second derivatives of f. . . . . . .
  • 132. Example (Continued) Let f(x, y) = x3 − 3xy2 . Find the second derivatives of f. Solution We have fxx = (3x2 − 3y2 )x = 6x fxy = (3x2 − 3y2 )y = −6y fyx = (−6xy)x = −6y fyy = (−6xy)y = −6x Notice that fyx = fxy , as predicted by Clairaut (everything is a polynomial here so there are no concerns about continuity). The fact that fxx = fyy is a coincidence. . . . . . .
  • 133. Partial Differential Equations Definition A partial differential equation (PDE) is a differential equation for a function of more than one variable. So the derivatives involved are partial derivatives. . . . . . .
  • 134. Partial Differential Equations Definition A partial differential equation (PDE) is a differential equation for a function of more than one variable. So the derivatives involved are partial derivatives. Example Let f(x, y) = x3 − 3xy2 . Show that f satisfies the Laplace equation ∂2f ∂2f + 2 =0 ∂ x2 ∂ y . . . . . .
  • 135. Partial Differential Equations Definition A partial differential equation (PDE) is a differential equation for a function of more than one variable. So the derivatives involved are partial derivatives. Example Let f(x, y) = x3 − 3xy2 . Show that f satisfies the Laplace equation ∂2f ∂2f + 2 =0 ∂ x2 ∂ y Solution We already showed that fxx = 6x and fyy = −6x. So fxx + fyy = 0. . . . . . .
  • 136. Tangent Planes and Linear Approximations I Learning objectives for Section 11.4 ◮ To understand the concept of a tangent plane to a surface z = f(x, y) and to be able to compute the equation of tangent planes ◮ To understand and be able to find a linear approximation to a function z = f(x, y). . . . . . .
  • 137. Tangent Planes and Linear Approximations II Learning objectives for Section 11.4 ◮ To understand the concept of a tangent plane to a parametrically defined surface r(u, v) = x(u, v) i + y(u, v) j + z(u, v) k and to be able to compute the equation of tangent planes. . . . . . .
  • 138. Tangent Planes and Linear Approximations III Learning objectives for Section 11.4 ◮ To understand and be able to find the differential to a function z = f(x, y), ∂z ∂z dz = dx + dy ∂x ∂y To be able to use the differential to estimate maximum error. . . . . . .
  • 139. Definition If f is a function and P0 = (x0 , y0 , z0 = f(x0 , y0 ) a point on its graph, then ◮ The linearization of f near (x0 , y0 ) is the function ∂f ∂f L(x, y) = f(x0 , y0 ) + (x0 , y0 )(x − x0 ) + (x0 , y0 )(y − y0 ) ∂x ∂y ◮ The tangent plane to the graph at P0 is the graph of L The linearization is the best possible linear approximation to a function at a point. . . . . . .
  • 140. Example Find an equation of the tangent plane to the surface z = y cos(x − y) at (2, 2, 2). . . . . . .
  • 141. Example Find an equation of the tangent plane to the surface z = y cos(x − y) at (2, 2, 2). Solution We have ∂z ∂z = −y sin(x − y), = cos(x − y) + y sin(x − y) ∂x ∂y So ∂z ∂z = 0, =1 ∂x (2,2) ∂y (2,2) Therefore the equation of the tangent plane is z = 2 + 0(x − 2) + 1(y − 2) = y . . . . . .
  • 142. Example The number of units of output per day at a factory is [ ]−1/2 1 −2 9 P(x, y) = 150 x + y−2 , 10 10 where x denotes capital investment (in units of $1000), and y denotes the total number of hours (in units of 10) the work force is employed per day. Suppose that currently, capital investment is $50,000 and the total number of working hours per day is 500. Estimate the change in output if capital investment is increased by $5000 and the number of working hours is decreased by 10 per day. . . . . . .
  • 143. Solution ( )[ ] ( ) ∂P 1 1 −2 9 −2 −3/2 −2 −3 (x, y) = 150 − x + y x ∂x 2 10 10 10 [ ] 1 −2 9 −2 −3/2 −3 = 15 x + y x 10 10 ∂P (50, 50) = 15 ∂x ( )[ ]− 3 / 2 ( ) ∂P 1 1 −2 9 9 (x, y) = 150 − x + y−2 (−2)y−3 ∂y 2 10 10 10 [ ] 1 −2 9 −2 −3/2 −3 = 15 x + y x 10 10 ∂P (50, 50) = 135 ∂y So the linear approximation is L = 7500 + 15(x − 50) + 135(y − 50) . . . . . .
  • 144. Solution, continued So the linear approximation is L = 7500 + 15(x − 50) + 135(y − 50) If ∆x = 5 and ∆y = −1, then L = 7500 + 15 · 5 + 135 · (−1) = 7440 The actual value is P(55, 49) ≈ 7427 13 So we are off by ≈ 1.75% 7427 . . . . . .
  • 145. Tangent planes to parametrized surfaces ◮ If r(u, v) parametrizes a surface S, and P0 = r(u0 , v0 ), then the vector r(u0 + h, v0 ) − r(u0 , v0 ) ru (u0 , v0 ) = lim h→0 h is tangent to S at P. ◮ If r(u, v) = x(u, v)i + y(u, v)j + z(u, v)k, then ∂x ∂y ∂z ru (u0 , v0 ) = (u0 , v0 )i + (u0 , v0 )j + (u0 , v0 )k ∂u ∂u ∂u ◮ We have another tangent vector ∂x ∂y ∂z rv (u0 , v0 ) = (u0 , v0 )i + (u0 , v0 )j + (u0 , v0 )k ∂v ∂v ∂v ◮ Their cross product is normal to the tangent plane. . . . . . .
  • 146. Example Find an equation of the tangent plane to the parametric surface r(u, v) = u2 i + 2u sin v j + u cos v k at the point u = 1 and v = 0. . . . . . .
  • 147. Example Find an equation of the tangent plane to the parametric surface r(u, v) = u2 i + 2u sin v j + u cos v k at the point u = 1 and v = 0. Solution The point we are finding the tangent plane at is r(1, 0) = ⟨1, 0, 1⟩. The tangent plane is spanned by the two vectors ru (1, 0) and rv (1, 0): ru (u, v) = ⟨2u, 2 sin v, cos v⟩ =⇒ ru (1, 0) = ⟨2, 0, 1⟩ rv (u, v) = ⟨0, 2u cos v, −u sin v⟩ =⇒ rv (1, 0) = ⟨0, 2, 0⟩ So a normal vector to the tangent plane is ru (1, 0) × rv (1, 0) = ⟨2, 0, 1⟩ × ⟨0, 2, 0⟩ = ⟨−2, 0, 4⟩ This means an equation for the tangent plane is −2(x − 1) + 4(z − 1) = 0. . . . . . .
  • 148. Differentials This is really just another way to express linear approximation. Define new variables dx = ∆x, dy = ∆y, dz. Then the equation for the tangent plane through (x0 , y0 , z0 = f(x0 , y0 )) is ∂f ∂f dz = dx + dy ∂x ∂y And the concept that this is a good linear approximation is expressed as ∆z = f(x + dx, y + dy) − f(x, y) ≈ dx when dx and dy are “small enough” . . . . . .
  • 149. Example If z = 5x2 + y2 and (x, y) changes from (1, 2) to (1.05, 2.1), compare the values of ∆z and dz. . . . . . .
  • 150. Example If z = 5x2 + y2 and (x, y) changes from (1, 2) to (1.05, 2.1), compare the values of ∆z and dz. Solution We have dz = 10x dx + 2y dy When x = 1, y = 2, dx = 0.05, and dy = 0.1, we get dz = 0.9. On the other hand z(1.05, 2.1) = 9.9225 So ∆z = 0.9225. The difference is 0.0225, or 2.4%. . . . . . .